eMedici- medicine and acute care Flashcards
Adrenal insufficiency states: describe px based on biochem
addison’s disease-
Adrenal insufficiency is a failure of the adrenal glands to produce adequate amounts of adrenocortical hormones. It can be primary, secondary, or tertiary.
Primary adrenal insufficiency (Addison disease)
- Primary adrenal insufficiency can be caused by abrupt destruction of the adrenal gland (acute adrenal insufficiency; e.g., due to massive adrenal hemorrhage) or by its gradual progressive destruction or atrophy (chronic adrenal insufficiency; e.g., due to autoimmune conditions, infection).
- Autoimmune adrenalitis: Most common cause
-
Infectious adrenalitis
- Tuberculosis: most common cause worldwide, but rare in the US
- CMV disease in immunosuppressed states (especially AIDS)
- Histoplasmosis
-
Adrenal hemorrhage [2][3]
- Sepsis: especially meningococcal sepsis (endotoxic shock) → hemorrhagic necrosis (Waterhouse-Friderichsen syndrome)
- Disseminated intravascular coagulation
- Infitrates
-
Impaired activity of enzymes that are responsible for cortisol synthesis
- Cortisol synthesis inhibitors (e.g., rifampin, fluconazole, phenytoin, ketoconazole): drug-induced adrenal insufficiency
- Secondary adrenal insufficiency
- Secondary adrenal insufficiency is caused by conditions that decrease ACTH production (impaired hypothalamic-pituitary-adrenal axis).
- Sudden discontinuation of chronic glucocorticoid therapy or stress (e.g., infection, trauma, surgery) during prolonged glucocorticoid therapy
- Tertiary adrenal insufficiency
- Tertiary adrenal insufficiency is caused by conditions that decrease CRH production.
adrenal insufficiency ( loss of corticol hormones- androgens, glucocorticoids, mineralocorticoids
Hypocortisolism
A state of decreased production of glucocorticoids (e.g., due to adrenal insufficiency). Can cause weakness, fatigue, depression, decreased appetite, weight loss, nausea, vomiting, diarrhea, and abdominal pain.
Hypoaldosteronism
A state of either decreased aldosterone production (e.g., adrenal insufficiency, hyporeninemic hypoaldosteronism) or resistance to aldosterone (e.g., administration of K+ sparing diuretics). Since aldosterone normally increases the reabsorption of Na+ and water and decreases the reabsorption of K+ and the excretion of H+, patients manifest with hyperkalemia, hypovolemic hyponatremia, and mild hyperchloremic metabolic acidosis
Hypoandrogenism
Adrenal hyperplasia ddx
The idiopathic enlargement of adrenal gland tissue with associated hormone overproduction.
Most commonly occurs bilaterally and manifests with increased serum concentrations of adrenal hormones.
-
THE THREE C’s- conns, cushings,chromo
- Can result in hyperaldosteronism (Conn syndrome) and
- hypercortisolism (Cushing syndrome).
- pheochromo
How would Conns present? Hyperaldosteronism
- Since aldosterone normally increases the reabsorption of Na+ and water and decreases the reabsorption of K+ and the excretion of H+, patients manifest with
- hypokalemia,
- hypervolemic hypernatremia, and
- mild hypochloremic metabolic alklosis
How would cushings present- HyperCortisol state
- cushings syndrome ( primary ACTH independent adernal hypreplasia)
- cushing disease (secondary ACTH dependent hyperplasia-pituitary microadenoma)
Cortisol causes-
- Mobilization of energy reserves, encourages gluconeogenesis: hypoglyceamia
- tonic dis-inhibition of ADH: free water retention-
“CUSHINGOID” is the acronym for side effects of corticosteroids: Cataracts, Ulcers, Striae/Skin thinning, Hypertension/Hirsutism/Hyperglycemia, Infections, Necrosis (of the femoral head), Glucose elevation, Osteoporosis/Obesity, Immunosuppression, Depression/Diabetes.
Patients with secondary hypercortisolism due to ectopic ACTH production may present with rapid onset of hypertension and hypokalemia without other typical features of Cushing syndrome.
Consider a diagnosis of hypercortisolism in patients who present with proximal muscle weakness, central obesity, thinning skin, weight gain, sleep disturbance, and/or depression.
SGL2-I - dapaglitlozin and warfarin. Which should be stopped preoperatively?
SGL2-I should be stopped. I’m stressed states, ie surgery/illness. Can cause euglyceamic ketoacidisis. Warfarin can be stopped 5 days prior to surgery. Need for Bridging therapy is dependent on VTE risk.
Adrenal hormones
- *Overview of the** hormones of the adrenal cortexFeaturesAldosteroneCortisolDehydroepiandrosterone (DHEA)Hormone class
- Mineralocorticoid
- Glucocorticoid
- Androgen
- *Production site**
- Zona glomerulosa
- Zona fasciculata
- Zona reticularis
- *Function**
- Blood pressure regulation: influences renal sodium and water reabsorption and potassium excretion
- Electrolyte homeostasis
- See “Mineralocorticoids.”
- Mobilization of energy reserves
- Immunosuppression
- Antiinflammatory
- See “Cortisol.”
- Substrate in estrogen and testosterone synthesis
- See “Androgens.”
- *Regulation of secretion**
- Renin-angiotensin-aldosterone system (RAAS)
- CRH → ↑ secretion of ACTH in the pituitary gland → ↑ secretion of glucocorticoids and androgens in the adrenal cortex
- *Associated disorders**
- Primary hyperaldosteronism
- Adrenal insufficiency
- Congenital adrenal hyperplasia
- Cushing syndrome
- Adrenal insufficiency
- Congenital adrenal hyperplasia
- Adrenal insufficiency
- Congenital adrenal hyperplasia
- Paraneoplastic syndrome → ↑ ACTH secretion → bilateral adrenal gland hyperplasia
-
Carcinomas include:
disorientated and mumbling. recently been started on sertraline for depression. Apart from her disorientation in time and space, the patient does not have any abnormal signs on physical examination. Her blood pressure is 140/80 mmHg and pulse rate 90/min. Her laboratory results are shown:
Serum:
Sodium 112 mmol/L (135 - 145 mmol/L)
Potassium 2.7 mmol/L (3.5 - 5.5 mmol/L)
Chloride 90 mmol/L (95 - 110 mmol/L)
Urine:
Osmolality 900 mOsm/kg (500-850 mOsm/kg)
Sodium 31 mmol/L (<20 mmol/L)
Which one of the following is the most likely diagnosis?
SSRIs can cause syndrome of inappropriate anti-diuretic hormone secretion.
syndrome of inappropriate anti-diuretic hormone secondary (SIADH) to recent commencement of sertraline; given the low serum sodium, confusion, altered mental state, high urine osmolality and high urine sodium.
In SIADH, anti-diuretic hormone is released in excess. This leads to increased AQP-2 in the apical membrane of the distal convoluted tubule and collecting duct of the nephron in the kidney, and increased resorption of water. This leads to high urine sodium and osmolality, and dilution of serum electrolytes (particularly sodium) with the reabsorbed water. This leads to hyponatremia, which is the cause of this patient’s confusion and altered mental state.
disorientated and mumbling. recently been started on sertraline for depression. Apart from her disorientation in time and space, the patient does not have any abnormal signs on physical examination. Her blood pressure is 140/80 mmHg and pulse rate 90/min. Her laboratory results are shown:
Serum:
Sodium 112 mmol/L (135 - 145 mmol/L)
Potassium 2.7 mmol/L (3.5 - 5.5 mmol/L)
Chloride 90 mmol/L (95 - 110 mmol/L)
Urine:
Osmolality 900 mOsm/kg (500-850 mOsm/kg)
Sodium 31 mmol/L (<20 mmol/L)
Which one of the following is the most likely diagnosis?
Although Addison’s disease can cause increased urine sodium and decreased serum sodium; this presents as an “Addisonian crisis” which results in severe dehydration, vomiting, and muscle pain. This patient has a normal blood pressure, and no history of Addison’s disease, thus, it is an unlikely cause of her presentation.
Signs and symptoms [19]
- Hypotension, shock
- Impaired consciousness, coma
- Fever
- Vomiting, diarrhea
- Severe abdominal pain (which can resemble peritonitis)
Diagnosis [12][19][32]
- Based on clinical suspicion: Maintain a low threshold for diagnosis in at-risk patients , especially those with shock refractory to fluids and/or vasopressors. [33]
- Point-of-care testing and routine laboratory studies
- Endocrine testing: Consider if the diagnosis is uncertain (e.g., first presentation).
- Random cortisol level [35][36]
- The sample should be drawn prior to hydrocortisone administration.
- ACTH stimulation test: diagnostic confirmation once the patient has been stabilized (see “Diagnostics”)
Consider adrenal crisis in patients with severe hypotension refractory to fluid resuscitation and/or vasopressors.
Adrenal crisis can be life-threatening, so treatment with high doses of hydrocortisone should be started immediately, without waiting for diagnostic confirmation of hypocortisolism!
Management [1][19]
- Empiric glucocorticoid replacement
- Hydrocortisone (preferred) [1][36]
- Prednisolone (alternative if hydrocortisone is unavailable)
- Dexamethasone (least preferred alternative if hydrocortisone is unavailable)
- Consider adding mineralocorticoid replacement, e.g., fludrocortisone (off-label) for the following: [31]
- Patients receiving glucocorticoids other than hydrocortisone
- Patients with septic shock [36][37]
-
Fluid resuscitation
- 1 L of normal (isotonic) saline in the first hour
- Further management should be guided by clinical response (see “Intravenous fluid therapy” and “Shock”).
- Hypoglycemia: IV dextrose, e.g., 50% dextrose
- Identify and treat underlying causes (e.g., sepsis).
- Consider higher-level monitoring, e.g., intensive care
The 5 S’s of adrenal crisis treatment are Salt (0.9% saline), Sugar (50% dextrose), Steroids (100 mg hydrocortisone IV once, then 200 mg over 24 hours), Support (normal saline to correct hypotension and electrolyte abnormalities), and Search (for the underlying disorder).
Best option for smoking cessation?
behavioral counseling, nicotine replacement therapy NRT : vareniciline most effective —: nicotine receptor Agonist Varenicline is the most effective, single-form pharmacotherapy when used in combination with behavioural support, and has been shown to more than double the chances of long term cessation. It is a nicotinic receptor partial agonist that relieves symptoms of craving and withdrawal. It is contraindicated in patients who are breastfeeding and pregnant. Varenicline has been associated with mood changes, depression, behavioural and sleep disturbance, abnormal dreams and suicidal ideation, however, no causal link has been established and it is safe to use in patients with stable mental illness, although monitoring should occur. For people with a creatinine clearance of less than 30 mL/min, start with varenicline 0.5 mg daily for 3 days and then a maximum of 1 mg daily. Avoid varenicline in patients with end-stage kidney disease (eTG). Buproprion was originally developed for use as an antidepressant, it significantly increases cessation rates, however is less effective than varenicline. It carries a risk of lowering the seizure threshold and thus is contraindicated in patients with a history of seizures, eating disorders and those taking monoamine oxidase inhibitors; while caution should be taken with other medications that may lower the seizure threshold.
Pre-op drugs. Amlidipine Atorvostatin Metformin/sitagliptin Which need to be stopped before and when
Amcodipine: can be continued if normotensive- not hypotensive.
Atorvostatin can be continued
metformin/sitagliptin- stopped on day of surgery when fasting starts
what is the detail of ASA scores- what is the utility in practise?
general idea of the functional status of a comorbid patient and thus the risk posed to them undergoing surgery. This allows a risk benefit discussion to be had regarding the surgery, especially when it is an elective procedure and may not be absolutely necessary. One way to help conceptualise the risk is based on a patient’s ASA score.
ASA 1: healthy patient, non-smoking and minimal alcohol
ASA 2: mild disease without functional limitation (smoker, social drinker, pregnant, BMI 30-40, well controlled DM/hypertension, mild lung disease)
ASA 3: systemic disease with some functional limitation (e.g. poorly controlled DM or hypertension, COPD, morbid obesity (>40), heart failure, end stage kidney disease on dialysis, MI >3 months ago)
ASA 4: severe systemic disease that is a threat to life (e.g. MI <3months ago, CVA, severe valve disease, sepsis, ARDS, end stage kidney disease without dialysis)
ASA 5: patient will not survive without surgery (ruptured aortic aneurysm, massive trauma, intracranial bleed, ischaemic bowel)
ASA 6: brain dead patient for organ donation
GCS
ABC- First secure the airway if GCS is less than or equal to 8.
dexamethasone for head injury?
While effective in decreasing intracranial pressure due to malignancy, dexamethasone is contraindicated in traumatic brain injury due to an association between increased all-cause mortality and corticosteroid administration post-traumatic brain injury.
large city hospital after a building site accident in which he fell head-first onto a concrete floor. His pulse is 50/min and blood pressure 160/80mmHg. He is breathing spontaneously but irregularly. In response to pain stimulus, he opens his eyes, mumbles incoherently, and flexes the stimulated limb. He has a haematoma over the right temple. His right pupil is larger than the left, and the direct and consensual light reflexes are normal in the left eye and absent in the right eye.
extradural haemorrhage that will require emergency neurosurgical intervention, especially since he exhibits all features of Cushing’s triad - bradycardia, hypertension, and irregular breathing.
However, the most pressing concern is his conscious state: his GCS is 8 (E2V2M4), and his airway is now at risk of compromise. As such, he requires immediate intubation to secure his airway, and to ensure adequate ventilation and oxygenation in order to prevent secondary brain injury due to hypoxia.
A burr hole is not the preferred operation for this patient. A craniectomy or craniotomy is more effective due to the size of the hole and higher efficacy in evacuating the bleed. Although urgent surgical intervention will likely be required in this case, it is not the immediate priority here.
ACD vs IDA
NB- key features
ACD: ferittin is high/N, transferin is low N
IDA: ferritin is LOW, transferrin is high/N
In IDA- key features?
Hypochromic (MCH) microcytic anaemia
Hepciden- low (in ACD high)
Low ferritn, high transferrin.
Haemoglobin 102g/L (115-165)
Mean Cell Volume 75fL (80-100)
Mean Cell Haemoglobin Concentration 24pg (27-32)
DDX for IDA?
Thalassaemia minor- hb normally dx at younger age
Carcinoma of the caecum- occult blood loss, few syptoms. expect in the elderly
Abnormal uterine bleeding- young women
Dietary deficiency of iron- look for well balanced/vegitarian/vegan
Hookworm infestation- more likely malabsorption. look for histamine/IgE
Carcinoma of the uterus
EG answers
Iron deficiency anaemia which is caused by either
- inadequate intake of iron,
- mal-absorption of iron, or
- loss or iron through blood loss or
- excessive use of iron.
Iron loss through abnormal, heavy, uterine bleeding and/or inadequate iron intake through diet are the two most common causes of iron deficiency in women of childbearing age group.
A detailed diet and gynaecological history should be obtained to evaluate iron deficiency in this age group.
She is less likely to have inadequate iron intake as she eats a balanced diet with meat and vegetables.
Carcinoma of the caecum (leading to gastrointestinal blood loss) is uncommon in young women but needs to be considered as a cause of iron deficiency if she complains of gastrointestinal symptoms, has a family history of early gastrointestinal cancer or there is no other good cause for iron deficiency identified.
Hookworm infestation, causing malabsorption, is possible but less likely.
Thalassemia minor is possible in this ethnic background but would have been diagnosed in prior blood tests.
eg 2 old women non specific fatigue
most likely, based on her age and lack of signs, is caecal cancer. With no visible bleeding, asymptomatic ‘occult’ chronic blood loss is common in gastrointestinal tract cancers especially right sided large bowel cancer.
Gastric cancer can and does lead to iron deficiency anaemia, but is less common than bowel cancer and is associated with more symptoms even if they are vague such as early satiety, unintentional weight loss, nausea or ‘dyspepsia’.
Uterine cancer could also cause iron deficiency anaemia, but again is more commonly associated with some signs such as visible loss of blood - spotting.
Thalassaemia is an important differential, especially considering her Greek ethnicity, but is unlikely if she has no history of anaemia previously and there is evidence of iron deficiency on her bloods.
Dietary iron deficiency could be contributing, but is unlikely to result in iron deficiency anaemia in a woman who is no longer menstruating and hence has no sources of regular blood loss.
This woman should urgently undergo endoscopy and colonoscopy, as well as receive treatment for her iron deficiency anaemia.
Learning Points
An older individual presenting with iron deficiency anaemia needs to be investigated for bowel cancer with endoscopy and colonoscopy
Iron deficiency anaemia causes a hypochromic microcytic anaemia
three month history of difficulty in swallowing. He has a history of gastro-oesophageal reflux disease (GORD) treated with 20mg pantoprazole daily.
He denies weight loss, regurgitation, odynophagia, heartburn or dyspnoea. He has a 40 pack-year smoking history with a smoker’s cough and consumes about three standard drinks of alcohol a day. The physical examination is unremarkable.
An endoscopy is performed, which shows extensive and circumferential ulcerative oesophagitis immediately above a small (3 cm) hiatus hernia. Biopsies confirm severe inflammatory changes only.
MX of hiatis hernis:
Sliding hiatal hernia
A condition in which the gastroesophageal junction and part of the stomach temporarily protrude through the esophageal hiatus in the diaphragm into the chest during swallowing. The gastric fundus remains below the diaphragm. Most common type of hiatal hernia.
Management of patients with sliding hiatal hernia
Conservative management
- Lifestyle modifications
- Proton pump inhibitors (PPIs) if symptoms of GERD occur
Surgery: laparoscopic/open fundoplication and hiatoplasty [2]
- Indications
- Persistence of symptoms despite conservative management
- Refusal or inability to take long-term PPIs
- Severe symptoms/complications of gastroesophageal reflux disease: bleeding, strictures, ulcerations
next point- any despesia, consider
Red flag features of dyspepsia
- Family history of gastrointestinal cancer
- Immunosuppression
- Clinically significant unintentional weight loss
- Anorexia
- Dysphagia, odynophagia
- Early satiety
- Persistent vomiting
- Noncardiac chest pain (specific for patients presenting with heartburn)
- Suspicion of gastrointestinal bleeding (e.g., unexplained iron deficiency anemia, melena)
Patients ≥ 60 years of age [1]
The following is applicable to patients ≥ 60 years of age with or without red flag features for dyspepsia.
- Refer for EGD to exclude neoplasia.
- Test for the presence of H. pylori during the endoscopy using biopsies and/or a rapid urease test.
- If a clear etiology is identified using EGD (e.g., visualization of peptic ulcer, gastritis), manage accordingly.
- If the etiology remains unclear after EGD, consider further testing based on clinical suspicion.
–
EGD- short and long segmenet. length? Mx of each
EGJ biopsie- low grade vs high grade dysplasia. Mx?
GERD MANAGEMENT
Biopsie:
- Short-segment (< 3 cm of columnar epithelium between Z line and GEJ)
- Long-segment (> 3 cm of columnar epithelium between Z line and GEJ): higher cancer risk
- Complications: esophageal adenocarcinoma (see “Esophageal cancer”)
-
Management and surveillance
-
PPI therapy [27]
- Consider if asymptomatic.
- Continue maintenance therapy long-term if symptomatic.
- Endoscopy with four-quadrant biopsies at every 2 cm of the suspicious area (salmon-colored mucosa)
- If no dysplasia: Repeat endoscopy every 3–5 years.
- If indefinite for dysplasia: Repeat endoscopy with biopsies after 3–6 months of optimized PPI therapy.
-
If low-grade dysplasia:
- Endoscopic therapy of mucosal irregularities-Usually including endoscopic mucosal resection and radiofrequency ablation
- Alternatively: surveillance every 6-12 months with biopsies every 1 cm
- If high-grade dysplasia: endoscopic treatment of mucosal irregularities, e.g., radiofrequency ablation
- Consider antireflux surgery or resection of the segment based on a specialist’s evaluation
-
PPI therapy [27]
three month history of difficulty in swallowing.
He has a 20-year history of gastro-oesophageal reflux disease treated with 40mg pantoprazole daily. He denies weight loss, regurgitation, odynophagia, heartburn or dyspnoea. He has a 40 pack-year smoking history, with a smoker’s cough and consumes about three standard drinks of alcohol a day.
The physical examination is unremarkable apart a husky voice and a non-tender 2cm mass in the left supraclavicular fossa.
Which one of the following is the most appropriate diagnostic investigation?
vs
72 y/o three-week history of a painless swelling in her neck. Upon further questioning, she describes feeling generally unwell over the last few months.
She has lost her appetite and unintentionally lost 10kg of weight. She reports a long-standing “smoker’s” cough with no changes in sputum. She does not report a medical history although admits to a forty-pack-year smoking history.
On examination, there is a non-tender 2 cm smooth swelling deep to the skin in the left supraclavicular fossa. There are no abnormalities detected on examination of her chest and abdomen.
Which one of the following is the most appropriate next step in management?
Endoscopy is a first-line investigation for the diagnosis and staging of Oesophageal Carcinoma
Long-term, chronic GORD can cause metaplastic changes in the lower oesophageal epithelium (known as Barrett’s Oesphagus), predisposing patients to oesophageal adenocarcinoma
–
carcinoma of the oesophagus, given his presenting complaint, history of GORD and smoking, and enlargement of Virchow’s node on physical examination.
An endoscopy is the most appropriate initial investigation, both in terms of a visual inspection of the upper digestive tract and the ability to obtain tissue for histological examination.
Although there is an increasing tendency for the initial examination in these patients to be a CT scan, this is probably a matter of expediency rather than logical clinical practice. In other words, although CT scan is necessary for staging of the cancer, and in the real world it is often easier to get a CT scan performed than wait for an endoscopy, endoscopy is the most appropriate diagnostic investigation.
Barium swallow may indicate the presence of an obstruction, but will not specifically diagnose oesophageal carcinoma, and will not allow for pathological diagnosis.
-→
eg 2 The correct answer is “chest x-ray”. The most likely diagnosis to fit this clinical scenario is a carcinoma of the lung. This is due to the combination of supraclavicular lump, significant systemic symptoms, cough, and smoking history. The swelling in her neck could well be a metastatic deposit. The easiest and most accessible investigation to confirm these clinical suspicions is to perform a chest X-ray. Depending on what this shows a CT of the chest and ultrasound-guided fine needle aspiration of the neck lump can be considered.
CT neck, chest, and abdomen is a reasonable investigation to consider following a chest x-ray.
Indirect laryngoscopy is not an appropriate initial investigation for this patient. Indirect laryngoscopies are useful for the diagnosis of head and neck cancers such as those of the larynx. This patient’s presentation is most likely explained by a carcinoma of the lung.
An ultrasound-guided biopsy may be considered following a chest x-ray.
Gastroscopy may be useful for diagnosing cancers of the stomach which may metastasise to the left supraclavicular lymph node. (Also known as Virchow’s node). This is a possible differential, however, carcinoma of the lung is more likely and a chest x-ray is a less invasive first investigation.
Learning Points
- Carcinomas of the lung may metastasise to the supraclavicular lymph nodes.
- Chest x-rays are an accessible and appropriate first-line investigation for suspected carcinoma of the lung.
Andexanet during surgery?
Andexanet is a reversal agent for factor Xa inhibitors (eg, rivaroxaban, apixaban), currently not available in Australia, and is intended for the reversal of acutely severe bleeding caused due to factor Xa inhibitors. This patient is currently not on any factor Xa inhibitors and does not require emergency anticoagulation reversal.
Andexanet during surgery?
Andexanet is a reversal agent for factor Xa inhibitors (eg, rivaroxaban, apixaban), currently not available in Australia, and is intended for the reversal of acutely severe bleeding caused due to factor Xa inhibitors. This patient is currently not on any factor Xa inhibitors and does not require emergency anticoagulation reversal.
when is Prothrombin complex concentrate (PCC) used?
Prothrombin complex concentrate (PCC) and fresh frozen plasma (FFP) before surgery are indicated for rapid warfarin reversal in patients who require emergency surgery or in patients with severe ongoing bleeding. PCC contains vitamin K-dependent clotting factors and helps reverse the anticoagulant effect caused by warfarin due to the inhibition of vitamin K-dependent factors (eg, factor II, VII, IX, and X). FFP is basically plasma that is acquired by removing the cellular components of the blood. Between the two, PCC is preferred for warfarin reversal since FFP is associated with an increased risk of volume overload.
Rx with Ticagrelor. Can patelet transfusion be used?
Platelets transfusion for patients taking ticagrelor is not useful since ticagrelor is a reversible ADP-receptor inhibitor and will likely bind to the receptors of the transfused platelets and render them inefficient.
PMR and GCA
This patient has a severe temporal headache and also has had stiffness in the shoulders and systemic symptoms (eg, fatigue) for the last few weeks.
GCA should be highly suspected in this patient.
Rx; Steroids immediately!
Giant cell arteritis (GCA) is systemic vasculitis that presents in patients aged >50 years and is characterized by systemic symptoms (eg, fever, weight loss, fatigue), temporal headache, jaw claudication, and visual pathologies (eg, amaurosis fugax, anterior ischaemic optic neuropathy). GCA is also associated with polymyalgia rheumatica that presents with pain and stiffness in the shoulders and the hips.
Generally, inflammatory markers (eg, ESR, CRP) are elevated in such patients. If GCA is highly suspected, patients should immediately be started on high-dose oral prednisolone (40-60 mg daily) without waiting for diagnostic confirmation because GCA is associated with a high risk of permanent vision loss and should be managed as soon as possible. Determining the route and dosage of glucocorticoids is shown in the table below.
Transfer to regional centre is not needed since GCA can be managed with glucocorticoids alone which are easily available in small rural hospitals. Although temporal artery biopsy is the confirmatory test for GCA, treatment should not be delayed for diagnostic confirmation because of the high risk of permanent vision loss.
history of numbness between his thighs and an inability to pass urine.
His recent medical history includes carcinoma of the prostate which is being treated with goserelin.
On examination there is an area of numbness in the inner thighs, extending into the perineum. Percussion of the abdomen shows an area of dullness extending three finger breadths above the pubis.
Which of the following is the most appropriate diagnostic investigation
Sudden onset of urinary retention and associated neurological symptoms must always raise the possibility of spinal cord compression.
diagnosis in this patient is cauda equina syndrome and this must be confirmed or excluded promptly. Cauda equina syndrome is a serious complication of severe compression of the descending lumbar and sacral nerve roots.
It generally presents with:
- bilateral radicular pain,
- asymmetric motor weakness,
- hyporeflexia, and bladder dysfunction.
This patient has sensory loss extending to the perineum and neurogenic bladder (evidence by dull percussion note above the pubis).
If the diagnosis is confirmed, the patient will require emergency surgical intervention and decompression. An MRI is the diagnostic modality of choice for the diagnosis of cauda equina syndrome. An MRI can be used to define the possible changes of spinal epidural haematoma, disc protrusion, degenerative canal stenosis, and/or epidural abscess.
Plain X-ray of the spine may be indicated when there is a suspicion of degenerative bone disease, fracture, or other bone pathologies. It is not a good diagnostic investigation to look for soft tissue pathologies or nerve compression pathologies.
Non-contrast CT scan is not good diagnostic imaging to visualise the nerve roots. It may be helpful in the diagnosis of bone disease. However, this patient has clinical features suggestive of radicular disease and non-contrast CT would be less yielding than an MRI.
CT myelogram is a more invasive test than MRI. It can identify nerve roots but MRI is preferred since an MRI can also rule out other possible differential diagnoses (eg, epidural abscess). A CT myelogram may be indicated if the patient has some contraindication to MRI (eg, electrical implanted devices).
A PET scan shows the metabolic activity at a cellular level. It may be helpful for assessing metastases due to cancer. However, a PET scan is not useful for diagnosing radicular diseases (eg, cauda equina syndrome).
cauda equina vs conus medullaris
- *Conus medullaris**
- Caudal end of the spinal cord consisting of upper and lower motor neurons
- Origin of S2–S5spinal nerves
- Ends at the L2 vertebral level (in adults)
- Innervates lower extremities, bowel, and bladder
- Tapered terminal end of spinal cord; directly attached to filum terminale
- *Cauda equina**
-
Anterior and dorsal nerve roots of spinal nerves (L2–Co1)
- Lumbar
- Sacral
- Coccygeal
- Innervates the lower limb, perineum, and pelvic organs
- Innervates the internal and external anal sphincter
- Provides parasympathetic innervation to the bladder
- Bundle of tapered nerve roots resembles a horse’s tail
- *Overview of** compressive spinal emergencies[25]Compressive myelopathiesCauda equina syndromeSpinal cord compressionConus medullaris syndromeEtiology
- Damage to or compression of the spinal cord at any level
- Apart from degenerative disk diseases, can also be caused by vertebral metastases, trauma (epidural hematoma, vertebral fracture), and epidural abscess
- Damage to or compression of the spinal cord at the vertebral level T12–L2, resulting in injury to the conus medullaris (sacral and coccygeal spinal segments) [26][27]
- Common causes include spondylolisthesis, tumors, and trauma (e.g., vertebral fracture).
- Damage to or compression of the cauda equina (nerve fibers L3–S5) located below L2
- Common causes include large posteromedial disk herniation, trauma, and tumors.
- *Onset**
- Variable, bilateral
- Sudden, bilateral
- Gradual, typically unilateral
Pain - Localized neck or back pain
- Lower back pain
- Less severe radicular pain
- Lower back pain
- Severe radicular pain
- *Motor symptoms**
- Bilateral paralysis below the affected level of the spinal cord
- Hyperreflexia
- Positive Babinski sign
- Symmetric, hyperreflexic distal paresis of lower limbs, possibly fasciculations
- Achilles reflex may be absent.
- Asymmetric, areflexic, flaccid paresis of the legs
- Muscle atrophy
- *Sensory symptoms**
- Loss or reduction of all sensation below the affected level of the spinal cord
- Symmetric bilateral perianal numbness
- Sensory dissociation
- Saddle anesthesia: lack of sensitivity in the dermatomes S3–S5, affecting the areas around the anus, genitalia, and inner thighs (may be asymmetric)
- Asymmetric unilateral numbness and/or paresthesia in lower limb dermatomes
- *Urogenital and rectal symptoms**
- Sphincter dysfunction with urinary or bowel urgency, retention, or incontinence
- Early onset of bladder and fecal incontinence
- Erectile dysfunction
- Late onset of urinary retention
- Change in bowel habits due to loss of anal sphincter control
- Decreased rectal tone or bulbocavernosus reflex
- Erectile dysfunction
Straight leg test
Straight leg raise tests
Straight leg raise tests are used to differentiate between neurological and musculoskeletal causes of lower limb pain.
Positive test (SLT- nn impingement– correclate with reflex weakness to localize)
Lasègue test (top): With the patient supine and the knee in extension, the examiner lifts the patient’s leg. The test is considered positive when lifting to an angle 30º–45º elicits radicular pain, i.e., shooting pain that radiates from the back along the posterior surface of the elevated leg (some sources quote other ranges, e.g., 30º–70º). A positive test suggests lumbosacral nerve root impingement.
Bragard test/sign (middle): Following a positive Lasègue test, the leg is slightly lowered (just enough for the pain to disappear) and the ankle dorsiflexed. If dorsiflexion produces pain, the Bragard test is considered positive. A positive Bragard strengthens the diagnosis of lumbosacral nerve root impingement.
Pseudo-Lasègue sign (bottom): A straight leg raise produces an aching pain along the back of the leg (rather than typical radicular pain). This indicates that the pain likely originates from the hamstring muscles (usually occurs with the leg flexed to > 60º). The Lasègue test is then interpreted as negative. A negative Bragard test helps differentiate the Pseudo-Lasègue sign from a true positive Lasègue test and further strengthens the suspicion of a muscular etiology.
Positive Lasègue test + positive Bragard test: neurological etiology likely
Pseudo-Lasègue sign + negative Bragard test: muscular etiology likely
reflex levels for each radiculopathy level
(L2 - S4)
- Overview of lumbosacral radiculopathies[9]**Radiculopathy**Causative diskSensory deficitsMotor deficitsReduction of reflexes*L3 radiculopathy**
- L2–L3
- Anterolateral area of the thigh
- Hip flexion
- Knee extension
- Hip adduction
- Adductor reflex
- Patellar reflex
- *L4 radiculopathy**
- L3–L4
- Anterolateral thigh, area over the patella, medial aspect of the leg, medial malleolus
- Knee extension
- Hip adduction
- Patellar reflex
- *L5 radiculopathy**
- L4–L5
- Lateral aspect of the thigh and knee, anterolateral aspect of the leg, dorsum of the foot, and the big toe
- Tibialis anterior muscle (foot dorsiflexion): difficulty heel walking (foot drop)
- Extensor hallucis longus muscle (first toe dorsiflexion)
- Posterior tibial reflex (medial hamstring)
- *S1 radiculopathy**
- L5–S1
- Dorsolateral aspect of thigh and leg, and the lateral aspect of the foot
- Peroneus longus and brevis muscle (foot eversion) and gastrocnemius muscle (foot plantarflexion): difficulty toe walking
- Achilles reflex
- Lateral hamstring reflex
- *S2 radiculopathy, S3 radiculopathy, S4 radiculopathy**
- S1–S4
- Posterior aspect of the thigh and leg (S2), perineum (S3–S4), perianal (S4)
- None
- Bulbocavernosus reflex
- Perineal reflex
X ray shoulder- dislocations
unconscious collapse at home. She has been unwell with a fever, rigors, and confusion over the last few days. On presentation today, her blood pressure has varied from 90/50 to 150/70mmHg, pulse rate 110/min, respiratory rate 22/min, and temperature 38.9 centigrade. She has rigidity in all four limbs, but normal reflexes and downgoing plantars. She is not rousable to voice. Her past medical history includes schizophrenia and bipolar disorder. She has taken risperidone for many years and was started on lithium nearly two weeks ago.
presenting with the distinctive clinical picture of neuroleptic malignant syndrome (NMS),
- characterised by muscle rigidity (often “lead-pipe” rigidity),
- normal or depressed reflexes,
- mental status change,
- and protracted onset (typically days as opposed to hours).
- Autonomic instability (fever, tachycardia, diaphoresis, labile BP) is also seen, however, is not specific to NMS.
- NMS will occur in the context of recent dopamine antagonist use e.g. antipsychotics or particular antiemetics.
- Investigations will also show an elevated CK, which can be used to distinguish NMS from similar clinical syndromes.
Her presentation could be solely related to her risperidone, as NMS can occur at any time during treatment (though typically the first week), however in the context of recently commencing lithium - a drug that may potentiate NMS with concurrent antipsychotic use - the new medication is the likely trigger. This is also consistent with lithium dosing as lithium typically takes 4 - 7 days to reach steady state concentrations, and her symptoms started during the second week of lithium therapy.
The most similar clinical syndromes to NMS are serotonin syndrome and lithium toxicity.
Serotonin syndrome can be distinguished by current use of serotonin agonist(s), more rapid onset within 24 hours, clonus, and hyperreflexia. It can also present with autonomic dysfunction, thus the fever, tachycardia, diaphoresis and labile blood pressure that can be seen in serotonin syndrome.
Lithium toxicity can be distinguished by supratherapeutic lithium intake or elevated serum levels, prominent gastrointestinal symptoms, and tremor.
Malignant hyperthermia is a condition that can result from the use of various anaesthetic agents and is associated with a high fever, tachycardia and muscle rigidity.
Whilst drug hypersensitivity can produce a similar picture of high fever and tachycardia, there is usually some form of cutaneous reaction (rash). Whilst all these conditions must be considered in the given clinical scenario, NMS is the most likely diagnosis and would explain all the given features.
It is important to note that in clinical practice multiple medications are often being taken by patients at varying dosages. As such, it is possible that patients may be experiencing manifestations from multiple clinical syndromes (eg. serotonin syndrome and NMS) concurrently.
Learning Points
Neuroleptic malignant syndrome (NMS) is caused by antipsychotics and characterised by fever, “lead-pipe” muscle rigidity, altered conscious state, autonomic dysfunction and elevated CK
drug of choice in the setting of head injuries and control of any subsequent seizures.
Traumatic brain injuries can be associated with the risk of subsequent seizures. Post-traumatic seizures can occur both in the early period (up to a week post-injury) and late period (after a week post-injury). The patient has risk factors for developing early PTS: Glasgow Coma Scale = 10, depressed skull fracture and penetrating head injury.
There has been a shift towards using levetiracetam for prophylactic pharmacological treatment due to its suitability for extended used and decreased side effect profile in comparison with phenytoin.
choice of beta blocker in an elderly patient with HFrEF
carvidalol, bisoprolol, misoprolol, Nebivolol, Atenolol, sotolo, Labetalol, Propranolol
four beta-blockers with evidence for use in congestive cardiac failure in terms of providing a mortality benefit are:
- metoprolol.
- nebivolol,
- bisoprolol,
- carvedilol,
These agents can reduce the mortality rate by about 30% over one year. Nebivolol has been studied for use in the elderly population with heart failure (SENIORS study).
The other three beta-blockers (ie, carvedilol, bisoprolol, metoprolol) provide significant mortality benefit but have not been studied extensively for use in the elderly population.
(AS) Atenolol and sotalol are associated with increased mortality in patients with congestive heart failure.
Labetalol is labeled for use in the acute management of severe hypertension or in hypertension during pregnancy. It is not indicated for use in congestive heart failure.
Although several studies have shown the mortality benefit of propranolol, it is not extensively studied enough to be considered a recommended choice of beta-blocker for heart failure.
SUMMARY
Use in HFrEF :Bisoprolol, Carvidalol, Misoprolol, Nebivolol. only Nebivolol in elderly
Atenolol, Sotolo (AS!)- increase mortality in HF
Labetalol, severe hyptns/ hypt in pregnancy (lullaby)
Propanalol- mortality- yes. HF?? not sure
X ray shoulder- dislocations
X ray shoulder- dislocations
A 50-year-old , reports five-month history of progressively worsening ankle oedema. exertional dyspnoea, accompanied by palpitations and a feeling of pressure in his chest when exerting himself.
drinks at least one bottle of wine daily. past medical history - type 2 diabetes and hypertension. currently taking metformin and perindopril.
he is haemodynamically stable.
On examination, the abdomen is not distended, soft and non-tender with no palpable masses.
His heart sounds are dual, with no murmurs and his pulse is regular. There is marked non-tender, pitting oedema of bilateral ankles. On auscultation of the chest, bibasilar crepitations.
Some investigations are performed.
Which of the following investigations would be most appropriate in determining the underlying cause of his peripheral oedema?
- Venous duplex ultrasound
- Echocardiography
- Liver ultrasound
- ECG
- 24-hour urine protein
Learning Points
Common causes of bilateral ankle oedema include heart failure, cirrhosis, renal failure and chronic venous insufficiency
This patient’s peripheral oedema has multiple possible aetiologies, including heart failure, renal failure and cirrhosis.
However congestive heart failure is most likely based on the combination of respiratory fluid overload as well as peripheral oedema. Therefore an echocardiogram would be most appropriate in determining the underlying aetiology and severity of the disease.
An ECG is a mainstay of assessment of any patient with shortness of breath, palpitations or chest pain, however, while arrhythmia may be a cause of cardiac failure, it is unlikely in this case to determine the underlying aetiology given that this patient’s pulse was regular. Furthermore, in this case the exertional palpitations are more likely to be secondary to heart failure.
Severe hypoalbuminaemia may contribute to oedema in context of liver cirrhosis or nephrotic syndrome, however given these disorders are less likely to induce pulmonary oedema than heart failure, a liver ultrasound or 24-urine protein are not likely to determine the cause in this case.
Peripheral venous insufficiency is the most common cause of chronic peripheral oedema, however, it does not explain the respiratory or cardiac symptoms this patient is experiencing.
A venous duplex ultrasound may be considered if there was a suspicion of venous insufficiency of DVT, although it is not appropriate in this case.
Causes of acute bilateral peripheral oedema include medication side effects (calcium channel blockers), or bilateral DVT.
While other chronic causes of bilateral oedema include lymphoedema or myxoedema - although neither are true oedematous states.
- Erythema induratum
- Erythema annulare centrifugum
- Erythema marginatum
- Erythema ab igne
- Livedo reticularis
Describe each
Erythema ab igne is a cutaneous reaction to repeated exposure of direct heat or infrared radiation to a person’s skin.
Heat sources that can cause this condition if there is repeated exposure include:
- Hot water bottles, heating blankets or heat pads to treat chronic pain
- Heated car seats, space heaters or fireplaces.
- Occupational hazards of silversmiths and jewellers (face exposed), bakers and chefs (arms, face exposed)
- Laptop computers rested on the anterior thighs
- Erythema ab igne has been reported in people with chronic pancreatitis, and is associated with hypothyroidism and
- lymphoedema.
Primary treatment of erythema ag igne is the removal of the causative heat source. Investigation of underlying causes of pain and excluding hypothyroidism should be considered.
Mild cases will settle spontaneously. In severe cases, abnormal pigmentation, atrophy or scarring may persist. Rarely, these lesions may develop into cutaneous malignancy. If there is any clinical suspicion of epidermal atypia, regular skin examination ± biopsies are recommended.
An important clinical differential for erythema ab igne is livedo reticularis. In livedo reticularis however, the skin changes are symmetrical (not focal to the point of heat source) and telangiectatic (rather than pigmented).
Erythema induratum (Bazin disease) is a panniculitis that typically presents on the calves, associated with a Mycobacterium tuberculosis infection.
Erythema annulare centrifugum is thought to be a chronic hypersensitivity reaction characterised by annular erythematous lesions, with a pathognomonic delicate scale behind the advancing edge (‘trailing scale’).
Erythema marginatum is a non-pruritic macular annular rash that typically occurs on the limbs which is associated with acute rheumatic fever.
Learning Points
Erythema ab igne is a cutaneous reaction to chronic direct heat exposure
Crohn’s disease
Pyoderma gangrenosum
Squamous cell carcinoma
Diabetic ulcers
Pseudomonas infection
systemically well.
On examination there are two large full-thickness ulcers across the right scalp. The ulcers have purple margins with an undermined edge. The surrounding skin appears normal. On the leg at the site of the previous ulcer, a cribriform scar was noted.
Swab mcs was unremarkable. The edge of one of the ulcers was biopsied and revealed a neutrophilic infiltrate with no evidence of infection or malignancy.
Pyoderma gangrenosum (PG) is a painful cutaneous ulcerative condition that is included in the group of neutrophilic dermatoses. It may affect any body site but does have a predilection for the lower limbs. It often develops at a site of minor tauma.
Characteristically the full-thickness ulcer has an undermined edge which is purple in colour. PG is not an infection, nor does it involve gangrene. Aetiology is poorly understood but is believed to be due to immune system dysregulation and genetic variations. Other neutrophilic dermatoses include Sweets syndrome, neutrophilic dermatosis of the dorsal hands, Behcet disease and bowel-associated-dermatitis-arthritis syndrome. PG is a diagnosis of exclusion - other causes of ulceration including infection, malignancy, vasculitis, diabetes and trauma need to be excluded.
Crohn’s disease can manifest as extra-gastrointestinal ulceration, however sites affected are commonly the trunk, arms and legs. Histology of cutaneous Crohn’s would reveala granulomatous infiltrate. Inflammatory bowel disease is associated with PG.
Cutaneous malignancy like squamous cell carcinoma is an important differential for chronic ulceration, particularly on the scalp.
Diabetes is a common cause of ulcers, typically occurring on the lower legs. Infection may be a cause of ulcers or occur within chronic ulceration and then impair healing. In this case, the clinical scenario points to the diagnosis of PG, with the histopathology excluding malignancy and wound swabs negative. If it was felt these conditions were clinically more likely, consider sampling error of investigations and repeat with representative samples.
Learning Points
Pyoderma gangrenosum is a painful cutaneous ulcerative condition that is a diagnosis of exclusion.
GLP 1 agonists and SGLT-2
GLP-1 agonists ( “Aglutide”) DOC with which co-existing Cardiovascular concern ()?
SGLT-2 DOC in (“flozin”)?
-
GLP-1 agonists and SGLT2 inhibitors have the best evidence base for cardiovascular benefits in patients with atherosclerotic ) cardiovascular disease
- (GLP-1, esp vascular problems)
- (SGLT-2, esp heart failure).
-
Considering renal function is important when selecting oral diabetic medications
- SGLT2 inhibitors require decent renal function to adequately cause a glucose lowering effect, and so in this patient with ESRF, this would not be an appropriate choice.
- SGLT-2 “iglaflozin”
- Dapagliflozin
- Empagliflozin
- Canagliflozin
S/E of keto acidosis- must be stopped prior to surgery
-
Contraindications
- Chronic kidney disease: If GFR decreases, drug efficacy decreases and adverse effects increase.
- Recurrent urinary tract infections (e.g., in patients with anatomical or functional anomalies of the urinary tract)
From stages 3–4 of chronic kidney disease, the side effects of the drugs outweigh the benefits.
Active agents
- incretins have “aglutide”
- semaglutide
- Exenatide
- Liraglutide- A rapid-release formula that is administered daily, liraglutide is also approved for the treatment of severe obesity with secondary complications
- Albiglutide- An extended-release formula that is administered once a week
- Dulaglutid
- GLP-1 antaganist
-
Clinical characteristics
- Glycemic efficacy: lowers HbA1c by 0.5–1.5% over 3 months
- Subcutaneous injection
- Weight loss (may be wanted)
- No risk of hypoglycemia
-
Side effects
- Gastrointestinal symptoms
- Nausea, vomiting
- Strong feeling of satiety (often desired)
- Pancreatitis and potentially pancreatic cancer [18]
- Potential risk of medullary thyroid cancer (MTC): further investigation required
- Gastrointestinal symptoms
-
Contraindications
- Preexisting symptomatic gastrointestinal motility disorders
- Chronic pancreatitis or a family history of pancreatic tumors
- Personal or family history of MTC or multiple endocrine neoplasia syndrome type 2 (MEN 2)
thiazolidinedione.- “glitzones” ass with advesre cardiovascular effects in HF
linagliptin
Gliclizide-Sulphomylurea, last line oral for T2DM
Gliclizide’ is a sulfonylurea, and would not be the best option for this patient. Although it is able to be used in atherosclerotic cardiovascular disease, it is not preferable to a GLP1-agonist, and is usually a last line oral option for management of T2DM.
‘Continuing with linagliptin’ is a potential option. The patient was already on this oral medication and so likely tolerates it well. However, the literature suggests that GLP-1 agonists are preferable in patients with atherosclerotic cardiovascular disease, due to their cardiovascular benefit.
DPP4 ingibitors “agliptin”
Active agents
- Sitagliptin
- Saxagliptin
- Linagliptin
Clinical profile [15][19][20]
- Mechanism of action: indirectly increase the endogenous incretin effect by inhibiting the DPP-4 that breaks down GLP-1 → ↑ insulin secretion, ↓ glucagon secretion, delayed gastric emptying
- Indications: See “Antihyperglycemic therapy algorithm for type 2 diabetes.”
-
Clinical characteristics
-
Important side effects
- Gastrointestinal symptoms: diarrhea, constipation (milder than in GLP-1 agonist exposure)
- Arthralgia
- ↑ Feeling of satiety (often favorable) due to delayed gastric emptying
- Nasopharyngitis and upper respiratory tract infection
- Urinary infections (mild)
- ↑ Risk of pancreatitis
- Worsening renal function, acute renal failure
- Headaches, dizziness
-
Contraindications
- Liver failure
-
Renal failure
- Sitagliptin and saxagliptin are not contraindicated in patients with moderate to severe renal failure, but a dose adjustment is required. However, sitagliptin in combination with metformin is contraindicated in patients with renal impairment because of the risk of lactic acidosis.
- Hypersensitivity
‘Rosiglitazone’ is not the best option in this patient. Rosiglitazone is a thiazolidinedione. It is associated with adverse cardiovascular outcomes, particularly heart failure, in susceptible patients. This would not be an ideal option for this patient.
- Favorable effect on lipid metabolism: ↓ triglyceride, ↓ LDL, ↑ HDL
- No risk of hypoglycemia
- Onset of action is delayed by several weeks.
-
Important side effects [11]
- ↑ Risk of heart failure
- ↑ Risk of bone fractures (osteoporosis)
- Fluid retention and edema
- Weight gain
- Rosiglitazone: ↑ risk of cardiovascular complications like cardiac infarction or death
-
Contraindications
- Congestive heart failure (NYHA III or IV)
- Liver failure
- Pioglitazone: history of bladder cancer or active bladder cancer; macrohematuria of unknown origin
GLP-1 antagonist (incretins have “-aglutide): atherosclerotic HD, with renal impairment
DPP4-I (‘-agliptins”)
SGLT-I (“-agliflozin”): cardiovascular dx with HF- NOT in renal imp
Metformin - DOC monotherapy, dual therap in T2DM- lactic acidosis. Careful with SGLT-1 and DPP4-I (both need decent renal function)- can precipitate severe lactic acidosis. Cant prescribe with SUs
Sulphonylureas (SUs)- (“mides,rides,nides”) last choice oral agents. Insulin secreation. Hypoglycemia
SU analogues -(a/eglinedes), Insulin secreation. Hypoglycemia
Thiazolidinediones (‘Glitzones”)- insulin sensitizers-adverse cardiovascular effects: precipitate HF
Alpha-glucosidase inhibitors- acrabose and maglitol
Glucagon-like peptide-1 receptor agonists (incretin mimetics)
Active agents
- Exenatide
- Liraglutide
- Albiglutide
- Dulaglutide
Dipeptidyl peptidase-4 inhibitors (gliptins)
Active agents
- Sitagliptin
- Saxagliptin
- Linagliptin
Sodium-glucose cotransporter 2 inhibitors (gliflozins)
Active agents
- Dapagliflozin
- Empagliflozin
- Canagliflozin
Sulfonylureas- (“amide, zide, ride”)
Active agents
-
First generation
- Chlorpropamide
- Tolbutamide
-
Second generation
- Glyburide (long-acting agent)
- Glipizide (short-acting agent)
- Glimepiride
insulin sectreation! The use of sulfonylureas in early stages of disease can lead to a premature loss of β-cell function, potentially impairing the effectiveness of antidiabetic treatment
Meglitinides (sulfonylurea analogue)
Active agents
- Repaglinide
- Nateglinide
Thiazolidinediones (glitazones, insulin sensitizers)
Active agents
- Pioglitazone
- Rosiglitazone
Alpha-glucosidase inhibitors
Active agents
- Acarbose
- Miglitol
- Particularly effective in controlling postprandial blood glucose levels. Because of reduced digestion of complex carbohydrates in the gastrointestinal tract, carbohydrates are digested by intestinal bacteria in the colon. This results in gastrointestinal side effects. Gradually titrating the dose normally minimizes these side effects.
-
Contraindications
- Severe renal failure
- Inflammatory bowel disease
- Conditions associated with malabsorption
Localisation of a parathyroid adenoma. imaging of choice-
Technetium-99m sestamibi scintigraphy has the highest positive predictive value and is the preferred primary investigation for the localisation of the adenoma.
Most common cause for primary hyperparathyroidism is a single parathyroid adenoma
Once the decision to operate is made, pre-operative radiological localisation is performed to facilitate minimally invasive surgery if the imaging is concordnant.
patient symptoms of generalised fatigue, weight gain and muscle weakness, in addition to insomnia and depressive symptoms are suggestive of Cushing syndrome, further supported by physical examination findings of high blood pressure, facial plethora, central obesity, easy bruising, and abdominal striae. Hypernatraemia and hypokalaemia are also commonly observed.
most appropriate test for cushings syndrome
midnight cortisol level
24-hour urinary free cortisol
High-dose dexamethasone suppression test
Plasma ACTH level
CT adrenal glands
ACTH stimulation test
Hypercortisolism has been demonstrated by at least two tests,
at least two cross-confirmatory results among three commonly-used tests:
- low-dose dexamethasone suppression test,
- 24-hour urinary free cortisol (ideally 3 separate collections), and/or
- midnight salivary cortisol.
Determine ACTH dependence (is ACTH-pituitary/PNPS, driving the hypercortisolism) -
If ACTH levels high -pituitary/ectopic ACTH production
If adrenal pathology (low ACTH- primary hypercortisolism (independent of ACTH))
A high-dose dexamethasone suppression test helps localise ACTH-dependent disease to the pituitary gland (Cushings disease).
CT adrenal glands is performed to characterise ACTH-independent disease (primary Cushings Sydrome) , which typically reflects either adrenal hyperplasia or a unilateral adrenal mass
ACTH stimulation TEST is used for ADDISONS DISEASE (adrenal insufficiency)
What is orlistat? When is it prescribed
Orlistat inhibits pancreatic lipases and prevents absorption of dietary fat, thus potentially resulting in steatorrhoea. This side effect may be improved by reducing dietary fat intake.
Reduced absorption of fat-soluble vitamins and steatorrhoea are common sequela to treatment with orlistat.
Otherwise well, A CT scan is performed and this is normal apart from an enhancing, homogenous 5cm mass in the region of the left adrenal gland. The margins are well defined and the lesion appears to be encapsulated.
Which one of the following is the most appropriate management plan?
Choose the single best answer.
- Repeat CT in six months
Urinary and serum hormone levels
CT-guided needle biopsy
Reassurance and no follow-up
MRI scan
‘adrenal incidentaloma.’ In otherwords, an adrenal lesion which is identified on imaging studies performed for other reasons, in the absence of directly suspected adrenal-related symptoms. The size of this lesion may indicate that intervention is necessary depending on further information. This is an adrenal adenoma and on size alone, needs to be removed to exclude malignancy. Any tumour greater than 4cm should be considered for excision.
A first step is to determine whether the lesion is functional or not. Hence, urinary and serum hormone levels is most appropriate at this stage, considering the possible diagnoses of
- Conn’s,
- Cushing’s or
- phaeochromocytoma.
Based on these results, if biopsy or surgery is indicated then further work-up may involve MRI to delineate soft tissue anatomy, or an image-guided biopsy.
It is crucial to know if the tumour is hormonally active before biopsy or surgery since certain adrenal lesions (e.g. pheochromocytoma) has very significant perioperative and intraoperative considerations for the surgeon and anaesthetist.
Reassurance alone is not appropriate, nor is a CT follow-up in 6 months.
What are the distingusging factors of Conns syndrome?
Primary hyperaldosteronism is characterized by hypokalemia and drug-resistant hypertension.
Primary hyperaldosteronism, sometimes referred to as Conn syndrome, is an excess of aldosterone caused by autonomous overproduction. It is typically due to adrenal hyperplasia (most commonly bilateral) or adrenal adenoma (typically unilateral).
Primary hyperaldosteronism is a common cause of secondary hypertension, occurring in > 5–12% of hypertensive patients.
High systemic aldosterone levels result in increased renal sodium reabsorption and potassium secretion, which lead to water retention and hypokalemia.
Patients are often asymptomatic and found to have hypertension at routine health checks. Hypertension due to primary hyperaldosteronism is often resistant to pharmacotherapy, and patients may have other signs suggestive of secondary hypertension, such as onset before the age of 30 or after the age of 55.
If symptoms are present, these are usually manifestations of hypokalemia (e.g., headache, muscle weakness, and polyuria).
Features of hypokalemia
- Fatigue
- Muscle weakness, cramping
- Headaches
-
Paresthesia in severe cases due to metabolic alkalosis
- Metabolic alkalosis decreases ionized calcium and presents with features of hypocalcemia.
-
Polyuria and polydipsia
- Mild nephrogenic diabetes insipidus
- Palpitations
- Constipation
Initial laboratory values in primary hyperaldosteronism classically show hypokalemia, metabolic alkalosis, high plasma aldosterone concentration (PAC), and low plasma renin activity (PRA).
The plasma aldosterone-to-renin ratio is the initial screening test, followed by confirmatory testing.
Further subtyping with imaging and/or adrenal venous sampling can determine whether aldosterone hypersecretion is unilateral or bilateral, which guides management. Treatment of unilateral disease consists of surgical resection, whereas bilateral disease is managed medically with aldosterone antagonists (e.g., spironolactone, eplerenone).
when to suspect pheochromo. Diagnostic test
Clinical presentation is related to fluctuating levels of excess epinephrine, norepinephrine, and dopamine secretion, therefore, symptoms are relapsing and remitting (episodic hyperadrenergic syndrome).
-
Episodic hypertension (or persistent hypertension in some cases) [3]
-
Paroxysmal [2]
- Throbbing headache
- Diaphoresis
- Heart palpitations, tachycardia
- Pallor
- Abdominal pain and nausea
- Anxiety
- Weight loss due to increased basal metabolism
- Hyperglycemia
- Signs of polycythemia, if EPO is secreted
Best initial test: plasma free metanephrines test [4]- Metanephrines are metabolites of catecholamines
- Examples: homovanillic acid, vanillylmandelic acid
- High sensitivity
Iodine supplements? when to give hyper or hypo thyroidism?
Hypothyroidism
. Grave’s disease that proves to be ineffective to antithyroid drugs. In addition, the Grave’s ophthalmopathy seems to be worsening with severe exophthalmos, new onset of diplopia and lid retraction. How to manage? ATD hasnt helped..
Options:
Total thyroidectomy/Radioiodine ablation/ ATD (Propylthiouricil, Carbimazole)/ B-blockers
RIA- option for graves. but can worsen enopthalmos. Total thyroidectomy- best option
A new ATD wont change enopthalmos
B-lockers just control hypthyroid sympathetic symptoms
The most definitive treatment in Grave’s disease, in this case, is a thyroidectomy. This patient is a young female that has no co-morbidities. She would be a fit candidate for surgery.
Radioactive iodine is also another definitive treatment of Grave’s disease. This treatment is useful for patients with high surgical risk or previous surgery irradiation of the neck. However, radioactive iodine may initially worsen the appearance and symptoms of her Grave’s ophthalmopathy. Hence, radioactive iodine is contraindicated in patients with severe graves ophthalmopathy.
Her antithyroid drug (carbimazole) has failed to achieve euthyroidism in this individual’s Grave’s disease. Switching one antithyroid drug to another (propylthiouracil) will most likely achieve similar ineffective results.
Beta-blockers only provide symptomatic relief for Grave’s disease and is not a definitive treatment.
Iodine supplements are used in hypothyroidism instead.
Fragility fracture with gynaecomastia in a tall man, ex reveals small testes with normal penis.
Diagnosis?
- Cirrhosis
- Klinefelter syndrome
- Kallmann syndrome
- Testicular malignancy
- Prolactinoma
Fragility fracture with gynaecomastia in a tall man is suggestive of Klinefelter syndrome. Klinefelter syndrome is the most frequent chromosome disorder in males, characterised by the phenotype if the tall man with relatively long legs, small testes and gynaecomastia. Hypogonadism usually first presents in early adulthood and progresses with ageing. Other important differentials for gynaecomastia include prolactinoma, which may present with headaches and visual changes, cirrhosis, and Kallmann syndrome.
Kinefelter
-
Clinical features: Testicular dysgenesis and subsequent testosterone deficiency manifest at the onset of puberty (symptoms are rare during childhood).
- Signs and symptoms of hypoandrogenism
- Eunuchoid growth pattern: tall, slim stature with long extremities (Growth plate closure is delayed )
- Gynecomastia
- Reduced facial and body hair
- Testicular atrophy [9]
- Reduced fertility and libido
- Frequent azoospermia
- Micropenis
- Osteoporosis (common feature in adulthood)
- Possible developmental delay
- Neurocognitive dysfunction (impaired executive function and memory, decreased intelligence)
- Language impairment (affects expression more than comprehension)
- Poor social skills
- Associated disorders
- Mitral valve prolapse
- Increased risk of breast and testicular cancer
- Metabolic syndrome
- Signs and symptoms of hypoandrogenism
Gonadal dysgenesis- female phenotype
Gonadal dysgenesis
Results in different degrees of hypogonadism and sex development
Typically results in the development of malfunctioning gonads (streak gonads) with fibrous tissue instead of normal germ cells, increasing the risk of malignancy (e.g., dysgerminoma, seminoma)
- Turner syndrome: 45,XO
- 46,XY gonadal dysgenesis
- 46,XX gonadal dysgenesis
Turners: Karyotype
- Meiotic nondisjunction (most often in paternal gametes) → complete sex chromosomal monosomy (45,XO; no Barr body)
- Mitotic nondisjunction of an embryonic cell → sex chromosomal mosaicism (45,XO/46,XX) → mild phenotypic expression
- Pathophysiology: chromosomal nondisjunction → X chromosome monosomy/mosaicism → impaired ovarian development → malfunctioning streak gonads with connective tissue instead of normal germ cells → estrogen and progesterone deficiencies
- Hypergonadotropic hypogonadism:
- Primary hypogonadism- A condition of insufficient sex steroid production in the gonads that leads to low serum concentrations of sex hormones (e.g., estrogen, testosterone) with a compensatory increase in the concentrations of pituitary gonadotropins (e.g., FSH, LH).
- ↓ estrogen, ↓ androgens, ↑ FSH, ↑ LH
- Karyotyping: confirmatory test
-
Treatment
- Estrogen and progestogen substitution
- Growth hormone (GH) therapy
- Surgical removal of streak gonads
Male phenotype D/O sexual development
Phenotypically male
- Kinefelter
- 46,XX testicular disorder of sex development
- Kallmann syndrome
Kallmann Syndrome (hypogonadotropic hypogonadism with hyposmia/anosmia)
- Prevalence: 3:100,000 male and 0.8:100,000 female population in the US [28]
- Karyotype: 46,XY or 46,XX
- Etiology: associated with more than 20 different gene mutations that result in the development of hypogonadotropic hypogonadism with hyposmia/anosmia
- ↓ GnRH → ↓ pituitary secretion of FSH and LH →
- *↓** testosterone in male individuals and ↓ estrogen in female individuals
-
Clinical features [29]
- Phenotype according to karyotype
- Anosmia or hyposmia
-
Infertility
- In male individuals: cryptorchidism and low sperm count
- In female individuals: primary amenorrhea
- Absent or attenuated pubertal changes (e.g., absent thelarche in female individuals, decreased growth spurt)
- In rare cases: neurological manifestations (e.g., ataxia, focal dystonia) [30]
- Associated disorders
Hyperthyroid states
- Thyrotoxicosis: a hypermetabolic condition caused by an inappropriately high level of circulating thyroid hormones irrespective of the source.
-
Hyperthyroidism: a condition characterized by the overproduction of thyroid hormones by the thyroid gland; can cause thyrotoxicosis
- Overt hyperthyroidism
- ↓ Serum TSH levels with ↑ serum free T4 and/or T3 levels
- Patients typically experience symptoms of thyrotoxicosis.
- Subclinical hyperthyroidism
- ↓ Serum TSH levels with normal serum free T4 and T3 levels
- Patients are normally asymptomatic or mildly symptomatic.
- May progress to overt hyperthyroidism
- Overt hyperthyroidism
Thyrotoxicosis: Can be due to overproduction by either the thyroid gland (i.e., hyperthyroidism) or an ectopic source, inappropriate release of thyroid hormone (e.g., in thyroiditis), or ingestion of exogenous thyroid hormone (accidental or intentional)
Hypothyroid States
- Overview of thyroid function tests in thyroid disordersEtiology****Thyroid function tests****TSH****levelFree T4*Free T3Overt hypothyroidismPrimary hypothyroidism
- Hashimoto thyroiditis (most common cause)
- Iatrogenic (e.g., following thyroidectomy or radioiodine therapy)
- Antithyroid medication (e.g., amiodarone, lithium)
- Transient hypothyroidism (e.g., silent thyroiditis, subacute granulomatous thyroiditis, postpartum thyroiditis)
- ↑
Secondary hypothyroidism - Pituitary disorders (e.g., pituitary adenoma)
- Infiltrative diseases
- Iatrogenic (e.g., following pituitary surgery)
- ↓
Tertiary hypothyroidism -
Hypothalamic disorders
Subclinical hypothyroidism - Asymptomatic
- Same etiology as primary hypothyroidism
- Mildly ↑
-
Normal
Euthyroid sick syndromeLow T3 syndrome - Occurs in severe illness or severe physical stress (most common in intensive care patients)
- Normal
- Normal
- ↓
Low T3 low T4 syndrome - ↓
Hyperthyroid (low TSH)- most common ddx
Graves
An autoimmune condition of the thyroid gland in which circulating TSH receptor autoantibodies lead to overstimulation of the thyroid gland and excess thyroid hormone production. Classic clinical signs include diffuse goiter, ophthalmopathy, and pretibial myxedema.
TMNG
Toxic multinodular goiter: Abbreviation: TMNG, Plummer disease
A multinodular enlargement of the thyroid gland with TSH-independent functioning of some nodules. It is the second most common cause of hyperthyroidism.
Toxic adenoma
A benign, solitary nodule in an otherwise nonpalpable thyroid gland. Caused by a gain-of-function mutation of the TSH receptor gene in a single precursor cell, which results in autonomous production of thyroid hormones
Postpartum thyroiditis
A lymphocytic thyroiditis that typically occurs 1-12 months after delivery and manifests with a period of transient thyrotoxicosis followed by hypothyroidism without thyroid enlargement or tenderness. Histologically characterized by lymphocytic infiltration and formation of germinal centers. Usually resolves spontaneously.
Subacute granulomatous thyroiditis Giant cell thyroiditis, De Quervain thyroiditis
A transient and self-resolving patchy inflammation of the thyroid gland that is associated with granuloma formation. Often occurs after a viral upper respiratory infection and is more common among women. The clinical course is typically triphasic, beginning with hyperthyroidism, followed by hypothyroidism, and finally a return to the euthyroid state. Classically presents with tender goiter, elevated ESR, and jaw pain.
Hyperthyroid (low TSH) approach to investigations
Anti-TRab, Throid scintography (RAI), US, MRI, TPO-ab
-
Initial evaluation: perform clinical assessment and screen thyroid function tests (TFTs) alongside routine laboratory studies.
- Consider alternate diagnoses if TFTs are normal.
- If characteristic features of Graves disease are present (see “Overview”): can stop investigations and begin empiric treatment
- Identify any tachyarrhythmia requiring treatment (see also “Atrial fibrillation” and “Acute management of tachycardia”).
-
Subsequent evaluation depends on the clinical picture . Options include:
- Thyroid scintigraphy: first-line for most patients with uncertain diagnoses, e.g., suspected thyroid adenoma or toxic MNG
- TSH receptor antibody (TRAb): for suspected Graves disease without characteristic features
- Thyroid ultrasound: first-line for pregnant/lactating patients, palpable nodules or suspected thyroiditis
-
Further evaluation: based on initial results
- Normal imaging and negative TRAb: consider serum thyroglobulin to identify exogenous thyrotoxicosis
- Inconclusive imaging and negative TRAb: consider thyroid peroxidase antibodies to identify thyroiditis
- Suspicious nodules visible on imaging: refer for FNAC
Subacute thyroiditis most commonly occurs after infection?
Viral infection. One of the only causes of hyperthyroidism to present with a sore t tender gland.
Anti-TRab, Throid scintography (RAI), US, MRI, TPO-ab
-
Initial evaluation: perform clinical assessment and screen thyroid function tests (TFTs) alongside routine laboratory studies.
- Consider alternate diagnoses if TFTs are normal.
- If characteristic features of Graves disease are present (see “Overview”): can stop investigations and begin empiric treatment
- Identify any tachyarrhythmia requiring treatment (see also “Atrial fibrillation” and “Acute management of tachycardia”).
-
Subsequent evaluation depends on the clinical picture . Options include:
- Thyroid scintigraphy: first-line for most patients with uncertain diagnoses, e.g., suspected thyroid adenoma or toxic MNG
- TSH receptor antibody (TRAb): for suspected Graves disease without characteristic features
- Thyroid ultrasound: first-line for pregnant/lactating patients, palpable nodules or suspected thyroiditis
-
Further evaluation: based on initial results
- Normal imaging and negative TRAb: consider serum thyroglobulin to identify exogenous thyrotoxicosis
- Inconclusive imaging and negative TRAb: consider thyroid peroxidase antibodies to identify thyroiditis
- Suspicious nodules visible on imaging: refer for FNAC
Thyroid scintigraphy [3][10]
- Indication: thyroid nodule(s) in a patient with low TSH
- Contraindications: pregnant and breastfeeding women
-
Findings and interpretation
- Cold nodule (hypofunctioning nodule): Evaluate for indications for FNAC of thyroid nodules with ultrasound.
-
Hot nodule (hyperfunctioning nodule): rarely malignant, FNAC not recommended [11]
- Solitary hot nodule: toxic adenoma
- Multiple hot nodules: toxic multinodular goiter
18-year-old woman is brought into the Emergency Department. Over the past three days, she has noted progressive weakness in the lower limbs and is now unable to mobilise. She was recently unwell with a prolonged course of diarrhoea three weeks ago. On examination, there is decreased tone, decreased power, and absent reflexes in her lower limbs. There is also loss of sensation in both her legs in a stocking distribution up to her mid-calf. Her pupils are equal and reactive, with a normal fundoscopy. Blood tests were unremarkable, except for a C-reactive protein 20mg/L [normal range <10mg/L].
Which one of the following investigations would provide the most diagnostic information when performed acutely?
This is a classical description of the clinical presentation and course of Guillain-Barré Syndrome (GBS). GBS often emerges after a preceding gastrointestinal or respiratory infection, as a result of molecular mimicry stimulating an immune response against peripheral nerves. Common precipitants include Campylobacter jejuni (which is suggested by the prolonged bout of diarrhoea she experienced), cytomegalovirus, Epstein-Barr virus and human immunodeficiency virus. Alternative triggers include immunisation, especially against influenza, as well as trauma and bone-marrow transplantation.
GBS presents with a progressive, symmetric muscle paresis +/- hyporeflexia +/- sensory and/or autonomic abnormalities, which often begins distally in the lower limbs.
Lumbar puncture (LP) needs to be performed in suspected GBS to exclude other infective or inflammatory causes of an acute flaccid paralysis. CSF analysis in GBS often reveals an elevated CSF protein with normal CSF white blood cell count, known as albuminocytologic dissociation. This investigation is the most pressing initial test to order.
neck masses
A 46-year-old woman presents with a painless swelling in her neck (image). It has been present for three weeks. She is otherwise in good health. The lump is non-tender and smooth-surfaced.
Ultrasonography demonstrated that it is cystic.
likely
Learning Points
- Thyroglossal cysts are the most common cause of midline neck masses in children
- A common way to establish the cause of a neck lump is its location
- Cystic hygromas are seen more commonly in paediatric populations
- Branchial cysts are commonly located in the upper third of the anterior aspect of the sternocleidomastoid muscle
The neck lump described and displayed in the image can be best identified by its location in the neck. It is sitting about two-thirds of the way up the anterior aspect of the sternocleidomastoid muscle. This is the typical site of a branchial cyst. These congenital abnormalities typically appear in the second or third decades of life and occasionally, as in this patient, can appear later. They are often asymptomatic but can become enlarged and tender if they become infected and frequently change in size during an upper respiratory tract infection.
Thyroglossal cysts are one of the most common causes of midline neck lumps. They are located more anterior-medially on the neck below the hyoid bone and are developed from remnants of the thyroid gland. They most commonly present during childhood between the years of 2-10.
Cystic hygromas are congenital malformations of the lymphatic ducts. They are also common in paediatric populations and are very rare to present in adulthood. They are commonly found in the posterior triangle of the neck.
An epidermoid cyst is derived from the upper portion of a hair follicle and are filled with keratin. They are typically 1-3cm in size (much smaller than the lump in the image) and can often be diagnosed clinically based on their appearance - firm, round, yellowish, fixed nodule.
Caseating necrosis is frequently seen in tuberculosis infections and other infectious diseases. It describes a form of cell death where the tissue forms a cheese-like appearance. It can infect the hilar lymph nodes, but would be rare to have the appearance shown in the image or this distribution of lymph nodes in the neck and the woman is asymptomatic in terms of an infectious point of view, hence is an unlikely diagnosis.
Painless jaundice. → check l.f.t.s → distinguish between hepatic and post hepatic
If g.g.t and alp is very elevated → post hepatic obstruction. Head of pancrease (painless, constitutional) V’s cholangitis (charcotstriad of r.u.q pain, fever, jaundice) → painless jaundice → head of pancrease.
A 69-year-old woman presents with a three-week history of anorexia and weight loss. Over the last three days she has noticed yellowing of her skin and eyes, dark urine and pale stools. She had recently returned from a holiday in Bali and had been taking doxycycline for prevention of malaria. She has no other past medical history and takes no regular medications. On examination she is haemodynamically stable, saturating well on room air and afebrile. She has significant jaundice affected her skin and sclera. She has multiple scratch marks across her arms and chest. Her abdomen in non-tender and murphy’s sign is negative. Bowel sounds are present and the liver span is of normal size. The liver function test results are: Bilirubin 293µmol/L (<25) Alkaline phosphatase (ALP) 1914U/L (40-130) Gamma glutamyl transpeptidase (GGT) 859U/L (<50) Serum alanine transaminase (ALT) 361U/L (<51) Serum albumin 31g/L (35-50)
Painless jaundice must always raise a suspicion for pancreatic cancer until proven otherwise. Furthermore, from this history the unexplained weight loss and anorexia hint at a potential malignancy diagnosis. Pancreatic cancer causes obstructive jaundice when the mass becomes large enough to obstruct the bile drainage, resulting in a post-hepatic cholestasis. All of the conditions listed could cause jaundice the biochemical picture is that of obstruction (very high GGT and ALP suggest post-hepatic obstruction) and that limits the likely diagnoses to one of the two post-hepatic problems; pancreatic cancer or cholangitis. But what makes pancreatic cancer more likely is her age and the painless nature. Cholangitis is acute infection of the bile duct. This presents with Charcot’s triad of abdominal pain, jaundice and fevers/chills. She does not display any of these symptoms besides the jaundice. Furthermore, despite her recent trip to Bali and potential exposure to viral hepatitis, she does not display the typical flu-like symptoms, fever and abdominal pain. Her LFTs also do not reflect an acute hepatitis picture, where the AST and ALT would be more raised. Similarly, a liver abscess may create abdominal pain or discomfort and deranged ALT and AST more than GGT and ALP. Doxycycline can cause hepatocellular damage and hence jaundice, but this is rare and often occurs in combination with other medications. Malaria can also cause jaundice, but she does not display the typical features of malaria and if she was taking doxycycline effectively she should have avoided this risk. Learning Points Painless jaundice must always cause concern for pancreatic cancer Cholestasis causes raised ALP and GGT more than AST and ALT Acute cholangitis can present with Charcot’s triad of jaundice, fevers and abdominal pain
A previously well 45-year-old man attends the Emergency Department after vomiting a small amount of fresh blood (less than half a cupful). One week prior to admission he had started a course of diclofenac for a back injury which he sustained while lifting a heavy weight at the gym. His pulse rate is 90/min, blood pressure on lying is 130/80 mmHg, with no postural drop. His haemoglobin is 135 g/L and his serum biochemistry results are all within the normal ranges. Which one of the following would be the most appropriate management plan? Choose the single best answer. Outpatient-based surveillance Admit and observe Admit for blood transfusion Urgent endoscopy Admit, transfuse and endoscopy the following day
Upper gastrointestinal bleeding is a common problem encountered in the Emergency Department. The goal of the initial assessment of a patient suffering from an upper gastrointestinal bleed is to determine the severity - thus allowing decisions to be made regarding management. A thorough history should be taken, particularly assessing for possible aetiologies, while an examination should focus on determining haemodynamic stability. For high-risk patients who are haemodynamically unstable, emergent management including fluid resuscitation and urgent endoscopy to achieve haemostasis. For haemodynamically stable patients, risk stratification tools such as the Glasglow Blatchford score (GBS) may be used to determine severity. This tool takes into account history features such as cardiac or hepatic past medical history, syncope and melaena, features of haemodynamic stability, and laboratory values of haemoglobin and urea. For a GBS greater than 1, an inpatient endoscopy is recommended. When the GBS for a patient is less than 1, then they are at low-risk and should be managed on an outpatient basis, for the benefit of the patient, and this hospital. According to the GBS screening tool, and his clinical stability, this patient does not need hospitalisation or urgent medical intervention. He is normally in good health, has no evidence of haemodynamic instability, has normal haematological and biochemical measurements and could be quite easily managed on an outpatient basis - provided he has supports at home and has access to medical assistance, should it be needed. An endoscopy can be arranged semi-electively to confirm the almost certain diagnosis of NSAID-induced ulceration. He should be sent home on a proton pump inhibitor and instructed not to take any more diclofenac. It is estimated that using the GBS tool, 15% of patients with an acute upper gastrointestinal haemorrhage can be classified as low-risk and managed safely on an outpatient basis.
Upper gi bleed in patient. What criteria/considerations to use to decide on Mx in patient/ out patient o urgent endoscopy.
Bus criteria. ① history of liver or cardiac disease. ② any syncope or maleana O ③ heamodynamically stable? ④ abnormal hb or urea. GBs >l endoscopy. If less than I patient can be managed safely in outpatient w endoscopic review tor cause. Discharge W psi.
Order of sequence for occult bleeding investigations.
If active bleeding: (ctimr). angiography It no active bleeding: 1 colonoscopy and endoscopy. Repeat if inconclusive. Lie do C and G scope twice. ② capsule endoscopy ③ if inconclusive enteroclysis (ct or MRI).
The patient has no gastrointestinal symptoms. The photograph shows the view as the procedure is about to start. The lesions are manually reducible. Assuming that the colonoscopy is otherwise normal, which one of the following is the most appropriate initial advice to give to the patient? Choose the single best answer. High fibre diet Sclerotherapy Banding Incision and drainage Haemorrhoidectomy
The view shows two partially prolapsed internal haemorrhoids, and given they are manually reducible, this classes them as Grade 3. The grading of haemorrhoids is as follows: Grade 1: Do not prolapse; located above the dentate line; often bleed Grade 2: Prolapse with straining; spontaneously reduce Grade 3: Prolapse with straining; only manually reduce Grade 4: Irreducible prolapse; may be thrombosed or ulcerated If the haemorrhoids remain asymptomatic, they should be able to be managed conservatively with a high fibre diet. Other conservative measure may include education regarding straining/lingering while passing stools, regular physical exercise, and avoiding medications and food that may cause diarrhoea or constipation. If mild symptoms develop, topical agents may be offered. If more troublesome symptoms develop, rubber band ligation should be offered as an outpatient. Banding is preferred to sclerotherapy as it has been shown to be more effective and require less treatment sessions. However, sclerotherapy should be offered to patients who are on antiplatelet or anticoagulation therapy as the bleeding risk is significantly reduced. Haemorrhoidectomy is reserved for patients with severely symptomatic Grade 3 or 4 disease. Learning Points Asymptomatic haemorrhoids can be managed conservatively with a high fibre diet Rubber band ligation is the procedure of choice for symptomatic disease
A 56-year-old man is being investigated for what he describes as a “volcanic-like” burning sensation in the lower chest after eating. This has got progressively worse over the last few years. An oesophago-gastroscopy is performed. The following was seen during the procedure. Which one of the following is the most likely diagnosis?
This man’s symptoms of retrosternal burning after eating indicate likely gastro-oesophageal reflux disease (GORD). Chronic GORD is one of the key risk factors for development of Barrett’s oesophagus, which is characterised by metaplastic change just proximal to the gastroesophageal junction (GOJ) where the normal squamous epithelium is replaced by intestinal columnar epithelium. In this case the areas of salmon-pink columnar epithelium can be seen extending some 3 - 4 cms above the cardia with tongues going up into the squamous epithelium. Barrett’s oesophagus can become dysplastic and progress to adenocarcinoma if left untreated. Biopsy results are necessary to confirm this diagnosis or indicate if it has become cancerous. Squamous cell carcinoma of the oesophagus tends to occur in the upper and middle portions of the oesophagus. These patients more typically present with dysphagia and often have a history of smoking and/or alcohol intake. These tumours can be difficult to identify on endoscopy during the early stages and may require staining with Lugol’s iodine to be visualised. Whilst this man’s symptoms are consistent with gastro-oesophageal reflux, there is no evidence of inflammatory change. Eosinophilic oesophagitis tends to occur in younger patients and is associated with dysphagia. Although a hiatal hernia may cause similar symptoms, it is diagnosed when there is >2cm proximal dislocation of the GOJ, i.e. the upper margin of the stomach folds can be seen above the diaphragmatic indentation.
Q1 -More definitive treatment of achalasia- grade 1 and 2
vs
Q2 -Manometric studies confirm type III achalasia. Extensive ( possibly with spasm )
most appropriate plan of management?
Botulinum toxin
Pneumatic dilatation
Cardiomyotomy and fundoplication
Peroral endoscopic myotomy
Oesophagectomy
Laparoscopic cardiomyotomy with fundoplication. With the development of high-resolution manometry, a more subtle distinction can be made between the various types of achalasia.
The Chicago classification of motility disorders describes three variants for achalasia, where Type I details the classical changes of a raised lower oesophageal sphincter pressure and no peristalsis in the body of the oesophagus. Treatment in this group of patients is aimed at reducing or abolishing the lower oesophageal sphincter, to allow food to ‘drain’ out of the oesophagus.
Botulinus toxin can produce good short-term results, but its durability is measured in months. Balloon dilatation can give effective long-term results - but can be complicated in about 15% of patients with troublesome gastro-oesophageal reflux. Another problem with balloon dilatation is the frequent need for repeated procedures and the risk of the patient then requiring surgery and the technical challenges from the resultant scar tissue.
Oesophagectomy is reserved for those with end-stage disease - and in selected patients, can give excellent relief of symptoms.
The peroral endoscopic myotomy (POEM) is a relatively new endoscopic approach to the division of the lower oesophageal sphincter and probably is most applicable to Chicago Type III cases (those with spastic contractions of the body of the oesophagus). Current experience with POEM suggests a 50% risk of long term proton pump inhibitor use to control gastro-oesophageal reflux symptoms brought about by the procedure.
A laparoscopic cardiomyotomy will produce good symptomatic relief of the troublesome dysphagia and regurgitation associated with this type of achalasia. The addition of a partial fundoplication will minimise any gastro-oesophageal reflux.
Botulinum toxin injection may provide immediate relief of symptoms, but its duration of action is measured in months. This form of treatment might be suitable for high-risk patients.
Pneumatic dilatation can produce effective relief of symptoms and keep patients in remission for 5 - 10 years. This treatment approach is most effective for patients with type II achalasia.
Laparoscopic cardiomyotomy can produce effective and sustained relief of symptoms in some 90% of patients and is most effective in young patients with types I or II achalasia and a tortuous oesophagus.
Peroral endoscopic myotomy (POEM) is the most recent treatment option for achalasia and is showing similar results to laparoscopic cardiomyotomy - but with a higher incidence of gastro-oesophageal reflux disease. With the long submucosal tunnel formed in a POEM procedure, the myotomy length exceeds that achieved at laparoscopic cardiomyotomy and is the preferred treatment for type III achalasia.
Oesophagectomy is reserved for those with end-stage achalasia where other treatments have failed. Despite the magnitude of the procedure, it can provide a major improvement to the patient’s overall quality of life.
UTI
most likely organism in UTI vs catheter associated UTI ( and urolithiasis present)
Whilst E. coli is the organism most usually associated with urinary tract infections
- *Learning Points**
- *Proteus mirabilis is a urea-splitting organism that promotes the formation of urinary tract stone**
the presence of urolithiasis in this case suggests that the likely responsible organism is Proteus mirabilis. This organism is a common cause of catheter-associated urinary infections. It is a Gram-negative rod, facultatively anaerobic, and produces the enzyme urease. This urease activity converts urea in the urine into ammonia, leading to alkalinisation of the urine, often with pH upwards of 8.0. A
lkalinised urine also facilitates magnesium ammonium phosphate precipitates (i.e. struvite stones) which can form kidney stones - which in this case was confirmed on the abdominal X-ray.
Klebsiella pneumoniae is another cause of catheter associated urinary tract infections, and is also associated with mildly alkaline urinary pH as it produces small amounts of urease, the enzyme responsible for the conversion of urea into ammonia. However, it typically does not present with urinary pH levels above 8 and is only associated with nephrolithiasis in rare cases.
Pseudomonas aeruginosa is also commonly associated with catheter associated infections, especially due to its ability to form biofilms that are resistant to antibiotic therapy. However, it does not cause alkaline urine nor does it cause nephrolithiasis.
S. saprophyticus is a common cause of urinary tract infections in young, sexually active women, but is uncommon in older and/or catheterised patients. Also, it does not cause alkalinisation of urine nor does it predispose towards the development of kidney stones. Additionally, staphylococcal species are unable to reduce nitrate to nitrite hence the urinary nitrite levels will be negative if it were to be a staphylococcal infection.
On examination, there is some tenderness over the L5-S1 vertebral foramina. The pain is reproducible with leg flexion, accompanied by abduction and external rotation…
what test is described and what does this indicate.
- buttock pain and worse with movements equivalent to the FABER test represents a classic presentation of sacroiliitis.
-
FABER test (Patrick test)
- Procedure: Flexion, ABduction, External Rotation
- Interpretation: The emergence of pain in the hip or sacroiliac joints indicates pathology in the corresponding structure.
- Procedure: Flexion, ABduction, External Rotation
-
FABER test (Patrick test)
Back pathology asscoiated with crohns disease
A 35-year-old man presents with a six-month history of pain in his upper buttocks. The pain is most troublesome at night, when it may awaken him from sleep, and in the morning as he feels stiff when he wakes up. He finds that as the day progresses the pain subsides and it also improves with exercise. The pain is not relieved by paracetamol and he was advised to avoid NSAIDs due to the risk of exacerbating his Crohn’s disease. He has no other symptoms. Crohn’s disease was diagnosed five years earlier and is well controlled with sulfasalazine.
On examination, there is some tenderness over the L5-S1 vertebral foramina. The pain is reproducible with leg flexion, accompanied by abduction and external rotation.
Which one of the following investigations is most appropriate?
- *C-reactive protein**
- *HLA-B27**
- *Plain X-ray of the lumbar-sacral region**
- *MRI sacroiliac joints**
- *Bone scan**
This patient’s presentation of inflammatory back pain, buttock pain and worse with movements equivalent to the FABER test represents a classic presentation of sacroiliitis.
Inflammatory back pain
A type of nontraumatic back pain that occurs secondary to inflammatory processes such as ankylosing spondylitis, psoriatic arthritis, reactive arthritis, and juvenile idiopathic arthritis. The pain most commonly affects the lumbar spine and is characteristically insidious in onset, worse at night or at rest, and relieved by activity. Affected individuals are typically < 40–45 years of age with raised ESR and CRP levels on laboratory studies.
Sacroiliitis occurs in 15% of patients with inflammatory bowel disease.
A plain X-ray is the first-line investigation, due to cost, availability and high-degree of clinical suspicion. A plain X-ray may show sacroiliac joint sclerosis in this patient. Ideally, this should be treated with exercise and/or non-steroidal inflammatory drugs (NSAIDs). As the patient cannot take NSAIDs, he should be encouraged to exercise and seek physiotherapist input.
C-reactive protein may be useful in determining if this patient is currently experiencing an exacerbation of inflammatory bowel disease, however, it is unlikely to be of any use in differentiating the cause of this pain.
The HLA-B27 gene is associated with rheumatological conditions such as ankylosing spondylitis (90-95%) and inflammatory bowel disease with sacroiliitis (50-60%), it may be present in this patient but does not aid in the diagnosis of his current presentation.
An MRI is the best imaging modality, and is able to detect early disease, however it carries a high cost and is dependent upon availability, and thus may be considered if an X-ray was negative. A bone scan is inappropriate due to relatively poor sensitivity and relatively high radiation load and thus is best avoided in a young patient.
Learning Points
Sacroiliitis is a common association of inflammatory bowel disease
FEV1 and FEV1/FVC in obstructive vs restrictive
(2) Obstructive lung diseases result in a concave flow-volume loop due to increased resistance. Absolute FEV1, as well as FEV1 relative to vital capacity (FEV1/FVC), is diminished. BOTH ARE DECREASED
(3) Restrictive lung diseases result in a narrowed flow-volume loop due to reduced vital capacity and compliance. Absolute FEV1 is diminished, but not in relation to vital capacity (FEV1/FVC).
© AMBOSS
How does parenchymal damage affect lung function.
Normal FEV1 with decreased VC, decreased RV, and decreased lung compliance is typically seen in patients with restrictive lung disease due to an intraparenchymal pathology (e.g., pulmonary fibrosis). Ankylosing spondylitis results in dyspnea due to restricted mobility of the thoracic cage and spine; the lung parenchyma itself remains unaffected.
Individuals with ankylosing spondylitis frequently develop some of the classical features of restrictive lung disease (i.e., normal/decreased FEV1 and decreased FVC) because of impaired mobility of the spine and thoracic cage. In contrast to most forms of restrictive lung diseases, lung involvement in ankylosing spondylitis manifests with a normal FRC and a normal/increased RV because the lung parenchyma, and therefore lung compliance, are unaffected.
Most other restrictive lung diseases are due to intraparenchymal lung pathology and manifest with decreased RV.
Treatment of ankylosis Spondaloaropathy
Etanercept
Additional treatment with a TNF-α inhibitor, such as etanercept, is indicated for improving spine mobility and reducing pain in patients with AS who have persistent symptoms despite NSAID use, which is the first-line treatment for ankylosing spondylitis, for > 2 months. Some studies suggest that these agents may also slow down radiographic disease progression after several years of treatment.
In addition, every patient should receive regular physical therapy to maintain range of motion and posture.
TNF inhibitors
A group of medications used to treat chronic inflammatory diseases such as rheumatoid arthritis, ankylosing spondylitis, and Crohn disease. Examples include etanercept, infliximab, and adalimumab.
among the DMARDs used in the treatment of AS, sulfasalazine is preferred over MTX.
MTX is used as first-line treatment of rheumatoid arthritis and psoriatic arthritis. RA of the spine can also result in back pain that improves throughout the day and is commonly associated with a positive family history. As for psoriatic arthritis, patients may also present at this age with spinal joint pain (in up to 40% of cases) and ankylosis on x-ray that may cause restricted movement. However, this man lacks other important findings of both conditions, such as arthritis of the phalangeal joints.
- *DMARDs such as leflunomid-**
- *Leflunomide** is used as first-line treatment of nonsevere psoriatic arthritis. Psoriatic arthritis may also manifest at this age with spinal joint pain (in up to 40% of cases) and ankylosis on x-ray that may cause restricted movement. However, this man lacks other important findings of psoriatic arthritis, such as arthritis of the phalangeal joints or nail changes (pitting, onycholysis).
Common associations of AS:
Common associations of AS:
Ankylosing spondylitis is a seronegative spondyloarthropathy and is associated with extraarticular manifestations, the most common of which is acute unilateral anterior uveitis (∼ 25% of cases).
Further findings in this patient would, therefore, most likely include eye redness and pain. Other associated, though rarer, extraarticular findings of ankylosing spondylitis include gastrointestinal (inflammatory bowel disease), cardiac (aortic insufficiency, AV block), and renal (IgA nephropathy).
Inferior vena caval obstruction
Portal vein thrombosis
Hepatic vein occlusion
Cirrhosis
Superior vena cava obstruction
The patient is thin, but has a massively distended abdomen. There is a collection of subcutaneous veins running around the umbilicus (also known as ‘Caput Medusa’ when localised at the umbilicus). The abdominal distension is due to ascites and the dilated veins reflect portal hypertension and the opening up of collateral vessels (umbilical vein) to carry blood back to the systemic circulation. The most common cause of portal hypertension is cirrhosis, mostly secondary to alcohol abuse in this patient. Chronic liver disease and cirrhosis results in many symptoms/signs which are easily examinable and candidates should have a clear approach to examination of these patients.
Inferior vena caval (IVC) obstruction results in blood pooling in the IVC causing a greatly decreased venous return to the heart and subsequent haemodynamic compromise; which is not present in this patient. In addition, any subcutaneous venous collaterals would extend from the lower to upper abdomen, as a reflection of dilatation of the inferior and superior epigastric vessels.
Portal vein thrombosis could lead to these physical signs but is less common than cirrhosis and does not fit the history as well. - in acute PVT more congestive symptoms (hepatorenal syndrome, fever oesophageal)
Hepatic vein occlusion (Budd-Chiari syndrome) is quite uncommon but could be suggested if the patient had pro-thrombotic risk factors such as a myeloproliferative disorder.
SVC obstruction commonly occurs with lung cancer externally compressing (possible in this patient with a smoking history), however it presents with oedema of the upper extremities/neck, chest pain and cough.
Pagets Disease- Clinical Features?
This patient’s hearing difficulties, bone pain, and tibial bowing raise suspicion for Paget disease of bone.
Bisphosphonates, which inhibit osteoclast-mediated bone resorption, are the first-line treatment for Paget disease of the bone.
Clinical features
- Approximately 70–90% of cases are asymptomatic.
- Bone pain, which may be associated with erythema and elevated skin temperature over the affected bones
- Pathological fractures: chalk-stick fractures of long bones [2]
- Bony deformities, e.g., bowing of legs (saber shin)
-
Skull involvement (in ∼ 40% of cases)
- Skull enlargement (increasing hat size)
- Cranial nerve deficits
- Impaired hearing: due to ankylosis of the ossicles and narrowing of the internal auditory meatus
- Headache
- Leonine facies
- Cauda equina syndrome, nerve root compression
GIST vs Gastric Malt on endoscopy
microcytic anaemia and an endoscopy is arranged. This is the view of the anterior wall of the body of the stomach.
The photograph shows a well-defined mass lesion on the anterior wall of the body of the stomach that is protruding into the lumen. The overlying mucosa is intact apart from ulceration in the middle of the lesion. This is the typical appearance of a gastrointestinal stromal tumour (GIST).
GISTs are a malignant mesenchymal tumour that arise from interstitial cells of Cajal in the gastrointestinal tract. 60% of these are found in the stomach, 35% in the small intestine and 5% elsewhere (colon, rectum, oesophagus or omentum). These tumours often remain asymptomatic and are found incidentally. Large (>2 cm) tumours can present in the 6th decade of life with anaemia and upper gastrointestinal bleeding.
Gastric MALT lymphomas frequently present as infiltrative, ulcerated lesions arising from the deep mucosa or submucosa.
Linitis plastica is a type of gastric cancer that commonly occurs in the fundus. It is characterised by thickening of the stomach wall and deformation of the stomach, leading to a ‘leather bottle-like’ appearance. It is not associated with discrete lesions, as shown in the photograph.
Type II neuroendocrine tumours are associated with Zollinger-Ellison syndrome, which is an acid hypersecretion state triggered by increased gastrin production. Endoscopy in this case would exhibit single or multiple polypoid nodules in the fundus and antrum, with multiple peptic ulcers.
Gastric varices tend to bleed quite severely, often presenting with frank haematemesis, haemodynamic instability (sometimes hypovolaemic shock) and abdominal pain. These often look like serpiginous vascular structures, and are associated with hepatic cirrhosis in the Australian population.
Learning Points
GISTs are most commonly found in the stomach.
Large GISTs can present with symptoms of anaemia and upper GI bleeding, but they are often found incidentally.
A 28-year-old woman presents with a three-month history of abdominal pain, fatigue, and diarrhoea which frequently contains blood and mucus. Her symptoms have worsened recently and she now has to defecate 5-6 times per day. She describes faecal urgency and an occasional feeling of incomplete defaecation. She does not have any recent travel history and has not taken any antibiotics recently. Her temperature is 37.2 C, blood pressure 110/75 mmHg, pulse 108/min and respiratory rate 18/min. On physical examination several erythematous painful nodules are noticed on the anterior aspect of her shins. Her blood tests are shown below:
Haemoglobin = 100 g/L (115-165 g/L) WBC = 13.0 x 109/L (3.6-11.0 x 109/L) Platelets = 450 x 109/L (140-400 x 109/L) ESR = 15 mm/hr (3-9 mm/hr)
A colonoscopy is arranged.
Which one of the following colonoscopic features would most likely be seen?
Choose the single best answer.
Erythema, mucosal friability, granularity and ulceration
Granulomatous inflammation on histology
Pale mucosa with petechial bleeding
Colonic biopsy showing inflammation extending to the muscularis
Hyperpigmentation of colonic epithelium
This patient’s bloody diarrhoea, faecal urgency, tenesmus and the erythema nodosum is most suggestive of ulcerative colitis.
Ulcerative colitis is an inflammatory bowel disease characterised by abdominal pain which may be diffuse or in the left lower quadrant, blood diarrhoea, faecal urgency, and tenesmus.
Ulcerative colitis is also associated with several extraintestinal symptoms (eg, erythema nodosum, uveitis, episcleritis, primary sclerosing cholangitis).
Patients may have anaemia due to blood loss, leucocytosis, thrombocytosis, and elevated inflammatory markers (eg, ESR). Platelets are acute-phase reactants and may be elevated in inflammatory conditions. Ulcerative colitis is difficult to differentiate from Crohn’s disease based on history and physical examination, and colonoscopy is indicated to confirm the diagnosis. The appearances of erythema, friability, granularity and ulceration - together with the clinical picture - would support a diagnosis of ulcerative colitis. Colonoscopic biopsy of the colonic epithelium in ulcerative colitis may reveal inflammation confined to mucosa and submucosa. A positive p-ANCA is characteristically associated with ulcerative colitis. It may also be associated with Crohn’s disease in 10-20% of the patients, however, it is more likely to be seen in patients with ulcerative colitis.
Inflammatory changes extending to the muscularis would be more in keeping with Crohn’s disease. Colonic biopsy of Crohn’s disease shows granulomatous (non-caseating) transmural inflammation involving all the mucosal layers of the intestinal wall. Crohn’s disease is more commonly associated with non-bloody diarrhoea. However, bloody diarrhoea may be seen in severe Crohn’s disease. p-ANCA is less commonly seen in patients with Crohn’s disease. Anti-Saccharomyces cerevisiae antibodies are more commonly associated with Crohn’s disease.
Pale mucosa with petechial bleeding is a characteristic finding in ischaemic colitis and is commonly seen in patients with hypotension or atherosclerosis. This patient does not have any recent history of vascular surgery and is unlikely to have atherosclerosis at this age. Moreover, elevated inflammatory markers support the diagnosis of inflammatory bowel disease.
Hyperpigmentation of colonic mucosa (Melanosis coli) is associated with laxative abuse and is more commonly seen in females in the healthcare profession (eg, nurse). Bloody diarrhoea, elevated inflammatory markers, and p-ANCA make this diagnosis unlikely.
Crohns vs Ulcerative Collitis
Crohn disease and ulcerative colitis Crohn disease
Pathophysiology
Frequency/type of defecation
Cr
- Increased
- Typically nonbloody, watery diarrhea
- May be bloody in more severe cases
UC-
- Greatly increased
- Bloody diarrhea with mucus
- Tenesmus
- Urgency
Nutritional status
- Cr-Poor or malnourished
- UC- Mostly normal, but weight loss and malnutrition may occur in severe disease [41]
Cr-
UC
- Painful defecation, pain located in LLQ
- Abdominal cramps and tenderness
- Tachycardia
- Orthostatic hypotension
Extraintestinal manifestations
Crones
- Nephrolithiasis (e.g., calcium oxalate)
- Cholelithiasis
UC
Both-
-
Skin
-
Eyes
- Mouth: aphthous stomatitis
-
Joints
- Peripheral arthritis
- Spondylitis
Crones - note fistulas are not ulcers
- Common (to skin, bladder, or in between loops)
- May cause pneumaturia and/or recurrent UTIs
UC- Rare for fistulas
Other complications-
Crones
- Abscess
- Strictures (obstructions)
- Perianal fissures
UC
Cancer risk
- Both:
- Increased due to underlying pathology
- Increased secondary to immunosuppression
Crones
U. Colitis
- Crones- ASCA, Anti-Saccharomyces cerevisiae antibodies
- UC: p-ANCA Myeloperoxidase antineutrophil cytoplasmic antibody
Endoscopy and imaging
Crones:
UC:
- Colon and rectum (exception: backwash ileitis)
Pattern of inflammation
- CR-Discontinuous (skip lesions)
- UC-Continuous
Typical diagnostic findings
- Cobblestone sign
- Linear and/or serpiginous ulcerations
- Small aphthous ulcerations
- Creeping fat
- String sign- A contrast-filled bowel segment that resembles a string on x-ray. Caused by marked narrowing of the bowel lumen, which may be caused by Crohn disease (usually stenotic terminal ileum), hypertrophic pyloric stenosis (elongated and narrowed pylorus), colon cancer, carcinoid tumor, or gastrointestinal tuberculosis
UC
- Friable mucosa- bleeds easily on colonoscopy
- Mucosal ulcerations can be deep or superficial
- Crypt abscesses- An infiltration of neutrophils into the lumen of intestinal crypts due to a breakdown of the crypt epithelium. Associated with ulcerative colitis.
- Loss of haustra (lead pipe sign)
- Cr-
- Transmural inflammation
- Noncaseating granulomas
- Giant cells
- Lymphoid aggregates (NON-non hodgkins lymphoma)
- UC-
- Confined to mucosa and submucosa (layers mucosa: epithelium, lamina propria (CT), muscularis mucosae. then submucosa) eg crypt abcess breaches lamina propria-→ still in mucosa
- No granulomas
Both have
- Neutrophilic inflammation of the crypts
TreatmentMedication
Crones- IL23 and Th17-secreates IL17 (fevers and wide spread)
- Corticosteroids
- Thiopurine analogs (azathioprine, 6-mercaptopurine)
- Anti-p40 antibodies (e.g., ustekinumab)
- Alpha 4 integrase inhibitors (e.g., natalizumab, vedolizumab)
- Possibly antibiotics outside of an acute episode (e.g., ciprofloxacin, metronidazole)
UC- Th2 (promotes B cells)
- 5-aminosalicylic acid (e.g., mesalamine)- down regulates immune response
- 6-mercaptopurine
- Calcineurin inhibitors (e.g., cyclosporine, tacrolimus)
Both
- Biologics (e.g., infliximab, adalimumab)
Food bolus obstruction and intermittent dysphagia in a younger patient
Food bolus obstruction and intermittent dysphagia in a younger patient- should raise the possibility of eosinophilic oesophagitis.
On endoscopy- concentric rings with linear bands.
First determine if ture EO or PPI responsive EO. 1 high dose PPI.
Repeact endo.
If unresponsive- true EO: Mx with concurrent elimination diet and high dose oral steroids (flucatisosne)
This is a typical history of a patient with eosinophilic oesophagitis and the endoscopy shows the characteristic concentric rings which occur secondary to eosinophil initiated oesophageal re-modelling and fibrosis.
Initially a 8-week course of high dose proton pump inhibitor (PPI), such as esomeprazole, is suggested followed by repeat endoscopy and biopsy to distinguish between patients with PPI-responsive oesophageal eosinophilia from true eosinophilic oesophagitis.
Once the diagnosis of true eosinophilic oesophagitis is confirmed then the patient can be commenced on concurrent elimination diet and oral ingested corticosteroids. Oral fluticasone has been shown to have good endoscopic and histological responses to treatment, with reduction in frequency of dysphagia.
The patient may need referral to an allergist. Rather than keeping patients on long-term PPIs, preference is now given to dietary modification.
When is SBP diagnosed? WHat is the treatment Empiric treatment of SBP.
Ascities more than 250 nuetrophills is diagnostic of SBP.
Patients with frank ascites, or those with cirrhotic liver disease at risk of ascites, should be managed with a high index of suspicion for spontaneous bacterial peritonitis (SBP) when presenting with abdominal pain or tenderness, fever, hypotension, or altered mental status.
Most likely organisms Enterobacteria or streptococcus- Empiric treatment needs Grm- and gram + cover ( Cephtriaxone cephotaxime)
- Whilst the combination of sofosbuvir and ribavirin is among the current first-line treatments for Hepatitis C based on genotype, the priority in this case is the spontaneous bacterial peritonitis.
- A therapeutic paracentesis might be helpful, but would not treat the underlying infection.
Note no anearobes in ascitiess- lots of O2
management of barrets oesophagus
On this occasion the presence of 3cm long segment of Barrett’s oesophagus is confirmed. There is a single 5mm area of irregularity and 4-quadrant biopsies are performed. The only abnormality is some high grade dysplasia in the irregular area. There is no evidence of oesophagitis or hiatus hernia.
Which one of the following is the most appropriate plan of management?
Choose the single best answer.
Endoscopic review in three months
Radiofrequency ablation
Endoscopic mucosal resection
Oesophagectomy
Neoadjuvant therapy followed by surgical resection
Q2:
three month history of dysphagia on a background of gastro-oesophageal reflux, the latter which has been present for many years. An endoscopy shows an area of ulceration with clear raised margins and surrounding inflammation extending up to the mid-oesophagus. The ulcerated area is biopsied and shows columnar-lined epithelium with areas of high-grade dysplasia.
Which one of the following is the most appropriate next step in management?
Choose the single best answer.
Repeat endoscopy and biopsies
pH and manometric studies
Esomeprazole and repeat endoscopy in three months
CT chest and abdomen
Argon ablation therapy
High grade dysplastic tissue can be difficult to distinguish from in-situ carcinoma and once identified must be actively treated. With the advances in endoscopic therapies, the current approach involves endoscopic mucosal resection (EMR) of the abnormal region and careful histological assessment of the resected tissues. This approach is all the more important if there are raised, irregular or nodular areas of Barrett’s oesophagus.
Radiofrequency ablation (RFA) may be used in combination with endoscopic resection and particularly for non-nodular areas of Barrett’s oesophagus. Further management will depend on accurate tumour (T) staging and possible use of endoscopic ultrasonography to assess local lymph node (N) status.
For high grade dysplasia alone, patients can be kept under endoscopic surveillance and where feasible, the entire segment of Barrett’s epithelium removed.
Oesophagectomy is indicated when cancerous cells have been identified and the tumour is considered resectable and beneficial to the patient’s prognosis (i.e. no known metastases).
-→ Q2
Learning Points
An area of ulceration in Barrett’s oesophagus must raise the concern of malignancy.
The presence of a well-defined ulcer and high grade dysplasia raises strong suspicions of oesophageal malignancy.
The length of the area of inflammation also increases the risk of malignant change.
Further endoscopy and biopsies of the ulcerated area must be done.
Patients who have biopsies showing high grade dysplasia should be referred to a tertiary specialist centre for repeat endoscopy, biopsy and potentially endoscopic therapy (endoscopic mucosal resection) or surgery.
Ablation therapy should only be done if frank malignancy is excluded. A single set of biopsies showing high grade dysplasia would not be sufficient.
This patient may require a proton pump inhibitor for control of their gastro-oesophageal reflux disease (GORD), however this can not be done in isolation. Esomeprazole alone is inappropriate without further investigation.
Manometric and ph studies are not helpful as GORD is already diagnosed.
Whilst a CT chest/abdomen will likely be done if the presence of malignancy is confirmed, the most important investigation is the repeat endoscopy and biopsy.
suspected TIA- how to decide on management: Asprin 300MG and Emergency center within 24 hours
Use risk stratifying for risk of stroke:
ABCD2
low (score: 1-4)- low
medium (score 4-5) - seen within 24 hours
high (score: 6-7)
This patient may have a transient ischaemic attack (TIA), which is supported by the transient focal neurological deficits e.g. right hemiplegia and transient vision loss.
According to the NICE guidelines and Australian Clinical Guidelines for Stroke Management 2010 by the NHMRC, individuals with TIA are given 300mg aspirin (loading dose) and referred for specialist assessment.
The ABCD2 score is a clinical predictor tool that risk stratifies an individual’s future stroke risk after a suspected TIA. IF suspeected TIA- ASPRIN AND neorovascular review and imaging in 24hours.
The ABCD2 score includes:
- Age ≥60 years = 1
- BP ≥ 140/90 = 1
- Changes
- Focal weakness = 2
- Speech disturbance without weakness = 1
-
Duration:
- Lasting ≥60min = 2
- Lasting 1–59min = 1
- Diabetes = 1
In this case, his ABCD2 score is 5 which is considered high risk. Individuals with ABCD2 score greater than 4 require an assessment within 24 hours.
suspected TIA imaging- high risk features
High-risk imaging findings:
Findings of high early stroke risk require rapid assessment and early secondary prevention measures.
High risk imaging findings include:
Distal vertebral and proximal basilar artery lesion -
Posterior circulation (vertebral, basilar, and posterior cerebral artery) ischemia is underdiagnosed because symptoms are often atypical (e.g., dizziness, ataxia, diplopia); however, the risk of early stroke is high and early revascularization may be indicated to prevent complications (e.g., locked-in syndrome).
Distal basilar artery and middle cerebral artery lesion - May portend an MCA stroke
Severe carotid artery stenosis - Lesions with ≥ 50% stenosis may benefit from early revascularization
Cerebral or cervical arterial dissection (see dissection of the carotid and the vertebral artery)- In young adults, dissection is an important cause of TIA to consider. Symptoms of arterial dissection may include head or neck pain, and the patient may benefit from endovascular repair
Shigella and Salmonella
Shigella and shigella toxin- bloody diarrhea. Only instance of dysentery where prescribe ABs to shorten duration- Floroquinalones - ciprofloxin or third gen cephalosporin.
Salmonella- thyphi and non thyphi. .
Salmonella Thyhi/para typhi– Typhoid/para typhoid fever: Constipated then diarrhea- possibly , Osteomelitis in SCD.
NO ABs- onli insever cases (floroquins)
Thyphoid/Paratyphoid fever
A systemic illness caused by by Salmonella enterica serotype Typhi/ Salmonella enterica serotypes Paratyphi A, B, and C.
Manifestations include fever, chills, abdominal pain, rose spots on the trunk and abdomen, and hepatosplenomegaly.
Salmonella non typhi (ie S typhrium)- Salmenellosis, diarrhea, possibly bloody. Reactive arthritis,
No ABS unless severe.
Salmonellosis-Salmonella gastroenteritis
A bacterial infection caused by Salmonella enteritidis and nontyphoidal salmonella. The second-most common cause of bacterial foodborne gastroenteritis. Associated with watery-bloody diarrhea, fever, chills, headache, myalgia, and severe vomiting.
Patient presents with abdominal pain, fever and positive S. non typhi stool culture. Then worsening abdominal pain.
what Complication of non-thyphoidal Salmonella looking for. What investigation.
Non-typhoidal Salmonella bacteraemia may occur as a result of intestinal salmonella infection.
A number of host factors predispose to salmonella bacteraemia including extremes of age, immunosuppression, chronic liver disease and haemoglobinopathies, as well alteration of the gastrointestinal tract by use of antibiotics, malnutrition or acid suppression.
Salmonella bacteraemia may result in a number of complications affecting a wide variety of sites including the urinary tract, lungs, pleura, heart as well as bones and joints.
An uncommon but important complication of salmonella bacteraemia is endovascular infection leading to a mycotic (or infected) aneurysm. Endovascular infections have been noted to occur in 10-20% of patients > 50 years old with Salmonella bacteraemia.
The classic presentation of an infected aneurysm is a painful, pulsatile, and enlarging mass with systemic features of infection. Salmonella typically targets the aorta, particularly if already diseased with atherosclerosis. The best diagnostic investigation for a suspected mycotic aneurysm is CT angiogram (MR angiography is an alternative if intravenous contrast is contraindicated).
A urogram may be a useful investigation if urinary tract colonisation was suspected.
A chest X-ray may be performed if there were respiratory symptom.
A CT brain and lumbar puncture may be useful if meningoencephalitis was suspected.
Learning Points
Aortitis is a rare but important complication of Salmonella bacteraemia
Hiatal hernias
six month history of heartburn and regurgitation. He is a non-smoker, walks 2km every day and goes to the gym regularly. He has not had any major illnesses in the past and does not use any medications. Physical examination is unremarkable. A contrast study is performed (see image).
appropriate plan of management?
Choose the single best answer.
- Esomeprazole
- Nifedipine
- Oesophageal dilatation
- Laparoscopic cardiomyotomy
- Laparoscopic fundoplication
Often linked to GORD- dont need s
The contrast study shows a small hiatus hernia with volume reflux, consistent with gastro-oesophageal reflux disease (GORD). There is no evidence of stricture formation. The most appropriate initial management for this patient is acid suppression with a proton pump inhibitor. The large majority of patients with GORD get good control of their symptoms with a proton pump inhibitor.
Calcium channel blockers (e.g. nifedipine) and laproscopic cardiomyotomy are management options for achalasia (which would be evident on barium swallow as a tapering of the inferior oesophagus).
Oesophageal dilation is only needed if stricture is present. The patient does not have any dysphagia, so there is no clinical nor radiological evidence of stricture formation. With the advent of effective acid suppression (proton pump inhibitors), stricture formation is now a relatively uncommon complication of gastro-oesophageal reflux disease.
Fundoplication is last line therapy for uncontrolled GORD and considering no treatment has been trialled so far, this approach would be inappropriate.
Patient with hematemesis. With strong Alchol use and child Pugh B cirrhosis. He is noted to have jaundice, splenomegaly and multiple spider naevi. Apart from a proton pump inhibitor and ceftriaxone, which one of the following medications is most strongly indicated? DOC in acute esophageal varices rupture. Choose the single best answer. Octreotide Somatostatin Propranolol Terlipressin Tranexamic acid
The correct answer to this question is terlipressin. This patient has a history of cirrhosis (likely secondary to chronic alcohol misuse) and presents with symptoms and signs suggestive of a massive variceal haemorrhage. He has not seen a gastroenterologist in over 3 years, so it is unlikely that he has had any recent screening endoscopies. Oesophageal varices arise due to portal hypertension, which leads to the development of porto-systemic collaterals. Rupture of these varices can result in severe and sudden blood loss, often proving fatal. In this case, the initial priority is to stabilise the patient, especially given his current haemodynamic compromise. This is achieved through the careful administration of crystalloid fluids (and blood products if required), as well as terlipressin, which helps to decompress the portal circulation and reduce blood flow through the porto-systemic collaterals. Therefore terlipressin is the correct answer. An IV antibiotic such as ceftriaxone and an IV PPI bolus are also indicated in variceal bleeding events. Octreotide and somatostatin are medications that can also be used to decompress the portal circulation, however they are both second-line behind terlipressin. Therefore, octreotide and somatostatin are incorrect. Propranolol is a non-cardioselective beta blocker, which is used longer-term to help prevent the development of oesophageal varices. As such, it does not have a major role in the acute management of variceal haemorrhage, making this answer incorrect. Tranexamic acid was previously thought to have had a role as an adjuct in the management of acute variceal bleeding, however a recent multi-centre international randomised placebo-controlled trial (HALT-IT) found no reduction in mortality from the administration of tranexamic acid in upper GI bleeding. As such, tranexamic acid is an incorrect answer.
Child B means ..l what criteria look at fro child score
haematemesis, where oesophageal varices were found to be the source of bleeding. Variceal banding was carried out, his condition stabilised and the patient is now being prepared for discharge home. Beyond the variceal banding, other measures will be required as part of secondary prophylaxis. Which one of the following management options would be most appropriate? Choose the single best answer. Metoprolol ACE inhibitor Propanolol Band ligation Terlipressin
Non-selective beta-blockers are the current drug of choice for primary prevention of variceal bleeding. Propanalol While metoprolol is also a beta-blocker, it is cardio-selective (B1 specific) and so does not confer benefit for prevention of variceal haemorrhage. Band ligation can be considered in patients where beta-blockers are contraindicated, or in those with imminent risk of rupture at endoscopy. Terlipressin is a vasopressor analogue that has a role in the management of variceal haemorrhage, however this is usually limited to after the bleeding has occurred and not as primary prophylaxis.